Nothing Special   »   [go: up one dir, main page]

Recalls 7 Compilation

Download as pdf or txt
Download as pdf or txt
You are on page 1of 117

RECALLS 7 COMPILATION 2. Darren’s intake during the shift is 360 ml.

RECALLS 7 - NP 1
3. Darren’s wound is 3 cm in length.
Situation: Quality documentation and reporting
are necessary to enhance efficient, 4. Darren’s wound is large and gaping.
individualization patient care. Nurse Tara is
assigned to Darren who is diagnosed with A. 1 and 3
diabetes mellitus for 10 years and has not be
compliant to his medications and diet. B. 2 and 3

C. 3 and 4

1. Nurse Tara is aware that a record should D. 1 and 4


contain descriptive, objective observations
about what the nurse sees, hears, feels and 3. Another guideline for good documentation

smells. The following describes what a record and reporting that Nurse Tara recalls is about

ought to be: completion. When Nurse Tara administers


medications, the following should be included,
EXCEPT:
A. Darren is uncooperative as manifested by his
refusal to take a bath
A. Any preliminary observations
B. Darren appears depressed.
B. Preliminary observations
C. Darren is anxious as manifested by his
tossing in bed and disarrayed beddings C. Patient’s response

D. Patient states, “I wish to end my life” D. Nursing measure taken for negative
response
2. When documenting the intake-output of
Darren, Nurse Tara should remember that she 4. Nurse Tara is making a discharge plan. The
must use precise measurements to ensure items she needs to document and report should
accuracy. Which of the following reflects include, EXCEPT:
accuracy?
A. Needs for referrals
1. Darren drank an adequate amount of fluid
during the shift. B. Nurses’ observations
C. Client’s involvement in the care plan 7. An important added data that Nurse Adam
should have if he is thinking of a urinary
D. Patient’s goals or expected outcomes problem is

5. Nurse Tara is ethically obligated to


A. Presence of blood in the urine
information about client’s illness and treatment
confidential. The person who has the legitimate
B. Presence of pain on urination
access to the patient’s record is the .
C. When the pain appears whether at the start
A. Husband or end of urination

B. Primary nurse D. Frequency, amount and burning sensation


on urination

C. attending doctor
8. An important added data that nurse Adam

D. patient should ask Esther, should we consider the


presence of a gynecological problem is
Situation: Nurse Adam is assigned in the
out-patient department. He admits Esther, 28 A. Date of menarche
years old, who is complaining of moderate to
severe abdominal pain over the right iliac region B. Date of last menstrual period
for the last 2 days. This is accompanied by
C. Her age and civil status
frequent urination.

D. Occupation

6. The FIRST STEP that nurse Adam should 9. An important added data that Nurse Adam

do is to . should ask Esther, should we consider the


existence of a surgical problem is

A. Percuss the abdomen


A. Fever
B. Get the vital signs
B. Vomiting
C. Palpate the right iliac region
C. constipation
D. Interview Esther
D. character of pain
10. There are two important laboratory commit unprofessional and unethical conduct in
examinations that could validate Nurse Adam’ the future?
presumptions on the case which are
A. Professional Regulatory Board of Nursing
A. CBC and stool examination
B. Philippine Nurses Association
B. Urinalysis and CBC
C. Professional Regulation Commission
C. Stool and urine examinations
D. Commission on Higher Education
D. Urinalysis and vaginal examinations
13. What Nursing Republic Act should Nurse
Situation: Nurse Leah has just passed the Leah refer too?
Nursing Licensure Examination. She needs to
refresh herself regarding the law that governs
A. RA 9371
the practice of nursing in the Philippines.
B. RA7164

C. RA 7664
11. By virtue of section 16 of the Philippine
Nursing Law, the first step she needs to do after D. RA 9173
passing the Nursing Licensure Examination is
to . 14. Since she needs to work in a hospital to get
hospital experience, which part of the Nursing

A. Take the oath of the profession before the Law should Leah focus for her safe practice as
Professional Regulatory Board of Nursing a novice nurse?

B. Apply for the Certificate of


A. Scope of Nursing Practice
Registration/Professional License

B. Nursing Research
C. Get an official copy of her board rating from
the PRC office
C. Nursing Education

D. Apply for a Professional Identification Card


D. Nursing Personal System

12. Which government body has the power to


revoke or suspend Leah’s certificate of 15. When Leah eventually gets employed in the
registration/professional license, should she hospital, she will initially be required to utilize
which of the following in rendering care to her A. Insomnia
clients?
B. Sleep deprivation related to problems in
school
A. Traditional and innovative approaches
C. Disturbed sleep pattern related to school
B. Therapeutic use of self problem

C. Health teachings D. Readiness for enhanced sleep

D. Nursing process 18. In order to help Ben go to sleep, what


simple and inexpensive nursing intervention
Situation: Ben, 15 years old, has been
can the nurse provide considering the type of
complaining of sleep disturbance for the past 3
room Ben is confined in?
weeks. He related that he feels weak and has
no vigor in doing his school work. He is brought
A. Room should be well ventilated
to hospital for specialization.

B. Light should be dimmed at specific time

C. Putting on the air conditioning unit


16. As his attending nurse, what will be the best
question to ask in order to find out Ben’s cause
D. Putting on dark-colored curtains
of sleep disturbance?
19. If Ben cannot sleep immediately, which of
A. “Could you tell me what you feel about the following can be provided by the nurse?
school work?”

A. A glass of warm milk


B. “Is there something or anyone who is
bothering you in school?”
B. A bedside story

C. “Could you share with me any major


problems that you have encountered in C. a sleeping pill
school?”
D. a warm bed bath
D. “What is bothering you in school?”
20. In terms of nursing care, the BEST help that
17. The nursing diagnosis appropriate for Ben the nurse can do is to .
is .
A. Avoid giving round the clock medications
B. Refrain from clutter at the nurse’s station 4. Administer Paracetamol every 4 hours

C. Omit vital sign tasking during the sleeping


5. Encourage wearing loose cotton clothing
time of Alex

D. Tone down nurse’s voice 6. Cover with thick blankets to stimulate


sweating
Situation: Simon, 66 years old, a foreigner,
was admitted to the hospital due to high fever
A.1,3,5
for five days. His temperature on admission was
37.5 ‘C with flushed and warm to touch skin. B.1,5,6
Nurse Celine, who is his attending nurse, gave
him a cool sponge bath. C.2,4,6

D. 2, 3, 4

21. Cool sponge bath is an independent 23. Client Simon wishes to know his present
intervention of nurse Celine aimed at . temperature in Farenheit. Nurse Celine’s
answer is which of the following?

A. Alleviating discomforts of client Simon


A. 102.2F
B. Generalizing full sponge bath care to client
Jimmy get hospital admission policy B. 101.3F

C. Diverting the temperature of client Simon C. 100.4F

D. Normalizing the vital signs of client Simon D. 99.5F

22. The independent nursing intervention to 24. Client Simon complains of severe chilly
reduce temperature of client Simon include sensation. What regulatory function of the
which of the following? Select all that apply. hypothalamus describes his reaction?

1. Monitor temperature regularly A. heat production

2. Offer only cold drinking water frequently B. heat promotion

C. heat adaptation
3. Provide tepid sponge bath
D. heat conversation A. 1, 2 & 3

25. After client Simon’s body temperature B. 1, 2 & 4


stabilized to normal, he began to complain of
feeling hot and finds difficulty of adjusting to the C. 2, 3 & 4

climate two days after. What is the


D. 3 & 4
APPROPRIATE answer to Nurse Celine?
27. Nurse Celine should prevent or reduce
A. “Foreigners cannot adjust to hot infectious agents PRIMARILY through the
temperatures” following practices. Select all that apply.

B. “People from cold regions have less brown


adipocytes” 1. Clean the room with antiseptic solutions

C. “Those who lived in cold places have lower 2. Proper garbage disposal
metabolic rate than those who live in tropical
region”
3. Mandatory hand washing upon entry to
D. “People from cold regions have more hospital
brown adipocytes”

26. Nurse Celine develops patient safety goals. 4. Improve safety administration of medicines

What nursing goal is APPROPRIATE for safety


environment? Select all that apply. A. 1, 2 & 3

B. 2, 3 & 4
1. Reduce the risk for patient harm resulting
from falls C. 1 & 3

2. Create a clean, orderly environment of D. 1 & 4

patient
28. Nurse Celine must comply with the policies
of health care facilities regarding isolation
3. Prevent high concentration drug errors
precautions. The BASIC precaution in the
hospital is _______________________.
4. Develop clean and simple signage for
directions
A. posting of signs of silence
B. limited visitors who are within the patient’s his hometown. He was just wearing a “sando”
age {sleeveless shirt} and short pants
when he was admitted to the hospital.
C. private room placement

D. strict utilization of personal things


31. What will be Nurse Chard’s PRIORITY
29. Nurse Celine includes air-borne precautions INTERVENTION?
in addition to Standard Precautions to be
observed at all times. These include, EXCEPT A. Put him near the electric fan to assist him in
_______. breathing

B. Ask him what triggered his attack this time


A. positioning and draping of patients

C. Place him in a comfortable environment


B. hand washing techniques

D. Put him near the nurse’s station so that he


C. wearing of masks and gloves could be assessed often

D. limited patient transport 32. What particular important vital sign should
Nurse Chard monitor and report to the
30. To protect spreading infection from one
patient to another, Nurse Celine must not forget physician?
to wash her hands _________.
A. Respiratory rate
A. before and after making rounds and
endorsement of all clients B. blood pressure

B. before and after performing procedures C. Cardiac rate


and in between patient care
D. Pulse rate
C. twice before and after all the procedures are
done to all the clients
33. Client Arvie knows that he will be given
decongestant through inhalation. What will be
D. apply alcohol before and after performing
procedures to all patients the instruction of Nurse Chard? That he should
breathe and that the nozzle of the NEBULIZER
Situation: Client Arvie, 33 years old, is known should be MAINLY near his _____________.
for his asthma attacks. His asthma attacks
commonly occur at the change of climate in A. Nose only
B. Face Situation: Aurora underwent mastectomy of
her right breast. After surgery, she was
C. Mouth only instructed to have range of motion exercises

D. Nose and mouth

34. Respiratory function is altered in clients with 36. What is the main purpose of this type of
asthmatic attack. Which of the following is the exercise for Aurora? It is to
cause of this alteration that Nurse Chard should ________________.
understand in order for him to make a good
nursing care plan?
A. prevent pain on the site of operation

A. Narrowing of the upper air passages B. prevent contracture deformity of the


muscle of the right arm and chest
B. Increased airway resistance
C. improve general circulation
C. Inadequate surfactant reaction
D. improve breathing
D. Paradoxical movements of the chest wall
37. The isotonic exercise such as Range
35. An important health teaching that Nurse of Motion exercises are those that are initiated
Chard must give before client Arvie’s discharge by the ________.
is ________.
A. patient
A. To avoid “sando” and shorts, if the weather is
cold B. physician

B. to use attire he is presently wearing only C. nurse


during summer time
D. relatives of the patient
C. to watch out for climate change and
unnecessary exposure to elements of the 38. Aurora needs some physical activity after
environment that trigger his asthma attacks
operation MAINLY to improve healing of which
parts of the body? Select all that apply.
D. to be careful always because constant
asthma attacks can trigger complication and
death 1. Skeletal 2. Integumentary
3. Respiratory 4. Muscular nursing practice in their work setting. A group of
nurses decided to conduct a study entitled,
A. 1 only “Effects of Zumba as a form of exercise in
promoting wellness among the health providers
B. 4 only in the hospital setting.”

C. 1 and 4

D. 2 and 3 41. Which of the following is the independent


variable in this study?
39. A good example of isotonic exercise for
Aurora’s arm are the following, EXCEPT
A. Exercise
___________

B. health care providers


A. combing the hair
C. zumba
B. brushing teeth
D. wellness
C. dressing up
42. Which is the dependent variable?
D. closing and opening hand

A. health provider
40. When doing a range of motion exercise, it is
important for the nurse to observe Aurora’s B. wellness
________.
C. exercise

A. Temperature
D. effects

B. Color of the skin


43. The research design of the study is

C. Blood pressure ____________

D. Respiratory rate A. quasi-experimental

Situation: One of the thrusts of management is B. experimental


to encourage more nurses to do research in
order to contribute to the improvement of the C. qualitative
D. descriptive that will lead to improve health. This approach
is popular because of the following reasons.
44. In the cause-and-effect relationship, which Select all that apply.
is the independent variable?

1. Views health as property of individuals


A. Population

2. Sees people as having the capacity to


B. Effect
change their lifestyle
C. Relationship
3. Assumes that if people do not act on
D. Cause
their health, they suffer the consequences

45. Which of the following is the appropriate


operational definition of wellness? Wellness is 4. Does not consider the social and

the state of well-being of the participants in environmental factors in which people live

Zumba on their ____________


A. 1, 2, 3

A. social stability
B. 2, 3, 4

B. spiritual growth
C. 3 and 4

C. physical health
D. 1 and 2

D. mental balance
47. Nurse Victoria finally plans to conduct

SITUATION: Some teachers in an elementary parent’s classes using the educational

school reported that a high proportion of approach. This approach is intended to _____.

children’s packed lunches contain hotdog or Select all that apply.

‘tocino’, sweetened drinks and potato chips.


School Nurse Victoria plans to conduct health 1. Provide information
education classes among parents on “How to
make healthy lunches and why it is important” 2. Develop the necessary skills

46. For this activity, Nurse Victoria considers


3. Provide change to a particular direction
using the behavior change approach to
encourage parents to adopt healthy behaviors
4. Help people make an impulsive choice 50. Since your participants are adult parents,
which of the following are appropriate
A. 1 and 2 strategies? Select all that apply.

B. 1, 2, and 3 1. Role playing

C. 2, 3, and 4
2. Group sharing
D. 2 and 3
3. Demonstration
48. To start the health education class and to
get the parent’s active participation, which of
4. Return demonstration
the following is the BEST question to ask?

A. 2 and 3
A. “Did you cook your child’s lunch today?”

B. 2 and 4
B. “Who among you have kids who are
underweight?”
C. 1, 2, 4

C. “Can you explain the “Plate Method”?”


D. 1, 2, 3

D. “What food do you usually prepare to


SITUATION: Safety in any health agency is a
your kids lunch?”
must. Nurse Amy wants this to maintain in her
49. The educational approach to health unit.
education states that learning involves three
aspects. Which of the following is NOT
included? 51. The nursing process facilitates an
understanding of the scope of challenges
A. Cognitive inherent in nursing care of clients at risk for
___________ EXCEPT:
B. Affective

C. Psychomotor A. Infection

D. Behavioral B. Suicide

C. Restraint
D. Injury SITUATION: Applying therapeutic
communication skills is vital in the nurse-client
52. The clients at risk require re-assessment of relationship.

their status on ____________

A. shift basis
55. Which of the following is the BEST
SEQUENCE of communication techniques to
B. daily basis
use during an assessment interview? Begin
C. as necessary basis with ____________.

D. injury A. giving a broad opening and move to


asking focused questions
53. When a patient is placed in a bed rest, the
nurse must watch which part of the body most? B. Giving information and move to asking
focused questions

A. Liver
C. Asking focused questions and close-ended
questions
B. Head

D. providing information and proceed to stating


C. Legs observations

D. skin 56. You are caring for Malou, sixteen years old,
suffering from acute leukemia. You want to
54. The Nursing Practice Standards which
actively listen to her concerns and understand
Nurse Amy must let her staff follow includes,
her meanings. You know that active listening is
EXCEPT ___________.
used to _______.

A. use a multidisciplinary approval to


enhance client safety as indicated A. Treat patient’s medical problems

B. implement emergency measures during fires B. Recognize the issue that is most
and disasters important to the client

C. risk elements should not be modified C. Help the patient become dependent to the
nurse in addressing his concerns

D. use infection control practices that prevent or


control transmission of the pathogens
D. Make conclusions regarding client’s SITUATION: Mr. BC is confines in the ICU in a
perception provincial hospital for some complications of his
type II diabetes. He is edematous and
57. You ask your newly admitted patient, “What
complained of severe chest pain. His vital signs
can we do to help you?” You know that this
are: BP160/98, temperature 37.2C; PR- 90
open-ended question will _____________.
bpm; RR- 30 bpm

A. result in specific information from the patient

B. allow patient to elaborate on his response 60. The order of the physician is for immediate
intubation. For the priority equipment, supplies
C. allow patient to briefly answer question
and material to be used for the procedure, the
nurse should collaborate with the _________.
D. put the patient at ease

58. As you listen to a patient, you need to A. operating room


provide feedback that is __________.
B. central supply unit

A. behavior-focused and evaluative


C. anesthesia department

B. well-timed and general


D. emergency department

C. general and content-focused


61. Morning care had to be done by the
________
D. well-timed and content-focused

59. Which of the following statements is TRUE A. nursing student


in relation to the use of humor?
B. all the options

A. Humor should focus on the client’s humanity


C. staff nurse

B. Humor is used to build rapport


D. nursing aide

C. Constant use of humor can be healing


62. In as much as you have not been trained in
initiating Intravenous infusion, who among the
D. Laughter increases neurotransmitters
following cannot also do it?
A. charge nurse pregnancy isconfirmed by the school physician.
Thea requested nurse Zenaida not to tell her
B. staff nurse parents about her being pregnant. She also
expressed desire to terminate her pregnancy.
C. nursing aide

D. physician

65. Which of the following ethical concepts


63. Turning Mr. BC every two hours would be
apply to patient Thea’s case?
difficult if you do it alone. In order to keep the
patient safe, the most number of nurse who can
A. Bioethics
team up with you will be ____________

B. Deontology
A. Four
C. Teleology
B. One
D. Intuitionism
C. Two
66. Nurse Zenaida should be guided by the
D. Three steps in helping Thea make an ethical decision.
Sequence the steps below.
64. If Mr. BC would be for dialysis and you need
to bring him to the dialysis unit, who among the
1. Gather data
following, could be the best to help you wheel
him to the area? A ______________.
2. Make decision

A. institutional worker
3. Identify and clarify the ethical problem
B. member of the family
4. Act and assess the outcomes of decisions
C. physician

D. nurse aide 5. Identify options or alternatives

SITUATION: Thea, 15 years old, and a A. 3, 1, 5, 2 & 4


high-school student, visited the clinic because
she suspects that she is pregnant. Her B. 1, 5, 2, 4 & 3
C. 1, 3, 5, 2 & 4 A. Autonomy

D. 2, 4, 1, 3 & 5 B. Justice

67. The INITIAL advice of nurse Zenaida to C. Veracity


patient Thea is which of the following?
D. Fidelity

A. Thea will inform her teacher about her


SITUATION: Emma, 65 years of age, has just
condition and to request the teacher to tell her
parents. been widowed a year ago. Her two children are
now all living in the U.S. She used to love
B. Thea will request the doctor to inform the cooking but since she is now living alone, she
parents
eats meal irregularly. Most of the time, she just
watches television. Her chief complaint is
C. Thea will choose a significant other close to
the parents to be the one to relay about her constipation.
pregnancy

D. Advise her that as a minor, her parents


should be duly informed about her 70. When assessing Emma, Nurse Alma should
pregnancy ask the following, EXCEPT:

68. Thea analyzed the advice of Nurse Zenaida


A. Dietary and fluid intake
and realized the value of life and family. This
attitude of realization is termed as B. Cultural beliefs
_____________
C. Lifestyle
A. Justice
D. Bowel pattern
B. Ethics
71. When auscultating the bowel sound of
C. Autonomy Emma, the nurse should be able to hear
___________.
D. Values clarification

A. loud and gurgling sound


69. Patient Thea’s decision regarding her
pregnancy is termed as _____________ B. increased bowel sound
C. decreased bowel sound D. 2, 3 & 4

D. absent sound 74. Health education should include which of


the following?
72. The MOST APPROPRIATE nursing
diagnosis for Emma’s case is
1. Responding to the urge to defecate and
_________________.
establishing a daily pattern

A. risk of constipation related to lifestyle


2. Role of dietary fiber and fluid in maintaining
bowel function
B. perceived constipation related to eating
habits
3. Role of exercise and activity in maintaining
C. dysfunctional gastrointestinal motility related
bowel function
to lifestyle

D. constipation related to inactivity 4. Safe and correct use of pharmaceutical


agents
73. Which of the following are the most simple
and cost effective interventions that the Nurse
A. 2, 3 and 4
can advise Emma to resolve constipation?
B. 1, 2, 3 and 4
1. Encourage increased fiber in diet
C. 2 and 3

2. Encourage physical activity and regular D. 1, 2 and 3


exercise
SITUATION: Obesity is an emerging concern
3. Regular time for elimination for patients and nurses. The nurses has an
important role in assessing and evaluating

4. Laxative patient’s physiological status in relation to wight


control

A. 1, 2, & 3

B. 3 and 4 75. Nurse Claire is assessing patients at the


outpatient clinic. Which of the following patients
C. 1, 3 & 4
is at risk for health complications related to 77. When working with an obese patient who is
weight? enrolled in a behavior modification program,
which nursing action is APPROPRIATE?
A. a thirty three year old who has a body mass
index (BMI) of 24 kg/m A. Having the patient write down the caloric
intake of each meal
B. A fifty six year old who is 6 ft (180 cm) tall
and weighs 150 lb (68 kg) B. suggesting that the patient has a reward
after achieving a weight loss goal
C. A seventy one year old who is 5 ft 4 inches
(160 cm), weighs 120 lb (55 kg), and carries C. encourage the patient to eat small
most of the weight in the thighs amounts throughout the day

D. a twenty four year old female with a waist D. asking the patient about situations that tend
measurement of 30 inches (75 cm) and a hip to increase appetite
measurement of 34 inches (85 cm)

78. When developing a weight-reduction plan


76. A patient who has been consistently
for an obese patient who is starting a weight
following a diet and exercise program and
loss program, which question is MOST
successfully losing one pound weekly for
important for Nurse Claire to ask?
several months is weighed at the clinic.
However, he has not lost any weight for the last
A. “What factors do you think led to your
month. Nurse Claire should first
obesity?”
_______________.
B. “Have you been on any previous diets?”
A. ask the patient whether there have been
any recent changes in exercise or diet C. “What kind of physical activities do you
patterns enjoy?”

B. discuss the possibility that the patient has D. “How long have you been overweight?”
reached a temporary weight loss plateau
79. An obese patient asks Nurse Claire about
C. Instruct the patient to weigh weekly and using Orlistat (Xenical) for weight reduction.
record the weights.
Nurse Claire advises the patient that
_________.
D. review the diet and exercise guideline with
the patient
A. drugs may be helpful in weight loss, but 81. To determine proper drug dosages for
weight gain is likely to recur unless changes children, calculations are MOST precisely made
in diet and exercise are maintained on the basis of the child’s __________.

B. the long-term effect of orlistat is not known, A. Weight


and the drug may cause serious side effects
such as heart valve problems B. Height

C. this drug can cause serious depletion of C. body surface area


fat-soluble vitamins and should be used only
several weeks
D. age

D. weight-reduction drugs of any type are used


82. Nurse Beth administered the intramuscular
for only those who do not have the will power to
reduce their intake of food medication of Iron by the z-tract method. This
method is used to ________________.
SITUATION: Nurse Beth is working on the
hospital’s pediatric unit. She is assigned as a A. provide more even absorption of the drug
medication and treatment nurse
B. provide faster absorption of the drug

C. prevent the drug from irritating sensitive


80. In preparing to give medications to a tissue
preschool-age child, which of the following
statements is an APPROPRIATE interaction by D. reduce discomfort from the needle

Nurse Beth?
83.The doctor ordered to give a one year old
patient an intramuscular injection. The
A. “Let me explain about the injection that you
appropriate and preferred muscle to select for
will be getting”
this child is the _______________.
B. “Do you want to take your medication
now?” A. Dorsogluteal

C. “Would you like the medication with water or B. Deltoid


juice?”

C. Ventrogluteal
D. “If you don’t take the medication now, you
will not get better.”
D. Vastus lateralis
SITUATION: Patient Charlotte, three years old, 86. In installing any drug into the ears, Nurse
was brought to the clinic by her mother due to Hazel performs in SEQUENCE. Which of the
ear ache and low-grade fever. While performing following steps?
her physical assessment, Nurse Hazel found
Patient Charlotte’s right ear to be inflamed and
1. Allow the drug to flow into the ears slowly
warm to touch. The Pediatrician ordered ear
drops to be instilled to the affected ear.
2. Tilt the head away from the nurse

3. Put a small cotton loosely into the ear


84. Nurse Hazel performs further physical
assessment on Patient Charlotte BEFORE drug
4. Wait for 15 minutes before instilling drops on
administration, which includes the following,
the other ear
EXCEPT _______________.

5. Instill the ear drops into the affected ear


A. Appearance of the pinna and meatus of the
ear
A. 1, 4, 2, 5, 3
B. Presence of the interference during the
drug administration B. 2, 5, 1, 3, 4

C. Type of any ear discharge C. 4, 1, 5, 3, 2

D. Location and extent of inflammation of the D. 3, 1, 4, 5, 2


ear
87. After instilling medications to the right ear,
85. The MOST APPROPRIATE nursing action
what is the MOST appropriate instruction to the
before instilling ear drops to Charlotte is to
mother?
______.

A. Remain in the position for 5 minutes


A. Check the medication to be within room
temperature
B. Pack a cotton pledge tightly to the ear

B. Refrigerate the medication for thirty minutes


C. Have the patient remain in the position for
one hour
C. Fill up the dropper with no more than one
millimeter
D. Advice the mother not to let Charlotte drink
hot water
D. Clean the outer surface of the dropper
88. In administering otic medication, Nurse D. Situational
Hazel
Situation: Nurses are expected to be leaders
A. Don the gloves making difference in the health care settings-
hospitals, clinics, communities and other
B. Perform hand hygiene
organizations.
C. Explain procedure and postpone

D. Check identification band and proceed


91. Why leadership plays a vital role in the
89. On the other hand, Nomar notices that the nursing profession?
Chief Nurse Executive has charismatic
leadership style. Which of the following A. It is not really important for nurses.
behaviors best describes this style?
B. Nurses have expert knowledge and are
interacting with and influencing the clients.
A. Possesses inspirational quality that
makes followers gets attracted of him and
C. Nurses should know how to direct people
regards him with reverence
towards accomplishment of goals.

B. Acts as he does because he expects that his


D. Nurses should always strive for higher
behavior will yield positive results
positions in the organization.

C. Uses visioning as the core of his leadership


92. Nomar understands that a Theory which

D. Matches his leadership style to the situation states that the Leadership style is dependent on
at hand. the situation. Which of the following styles best
fits a situation when the followers are
90. One leadership theory states that “leaders self-directed, highly motivated, experts and
are born and not made,” which refers to which matured individuals?
of the following theories?

A. Participative
A. Trait
B. Directive
B. Charismatic
C. Permissive
C. Great Man
D. Bureaucratic
93. Nomar would like to be a Transformational C. Organizational
leader. Which of the following statements best
describes this type of leadership? D. Dysfunctional

Situation: Nurse Kat is the head nurse of the


A. Uses visioning as the essence of
male medical ward in a tertiary specialized
leadership.
hospital handling multiple cases in her unit. It is
B. Serves the followers rather than being imperative that she is equipped with the basic
served. managerial skills.

C. Maintains full trust and confidence in the


subordinates
96. In this conflict resolution method, a person
D. Possesses innate charisma that makes ignores his or her own feelings about an issue
others feel good in his presence.
in order to agree with the other side.

94. Grace RN is a head nurse assigned at the


Intensive Care Unit of a Secondary Level A. Collaborating
Hospital. As a manager, Grace is expected to
B. Confronting
display effective conflict resolution skills. Which
of the following is true about conflict? C. Accommodating

A. It seldom occurs as part of the change D. Withdrawing


process in health care settings.
97. With this method of conflict resolution, each
B. It highlights differences in values, belief, side gives up something as well as gets
or actions.
something:

C. t is automatically negative
A. Negotiating
D. It discourages creativity and innovation.
B. Competing
95. Conflict that occurs between groups or
teams is called: C. Avoiding

D. Compromising
A. Interpersonal
98. Nurse Kat feels uncomfortable believing
B. Intrapersonal that she is the scapegoat of everything that
goes wrong in her department. Which of the D. They generally do not assume another
following is the best action that she must take? person’s point of view.

A. Identify the source of the conflict and RECALLS 7 - NP2


understand the points of friction
Situation
B. Disregard what she feels and continue to
work independently The family of Roxas is fond of dogs. A vendor
who entered the gate
without notice is bitten by one of the pet dogs
C. Seek help from the Director of Nursing
named Bert. PHN Cords Attends
to the vendor.
D. Quit her job and look for another
employment. 1. Which of the part of body of the vendor will
be the MOST affected in terms of rabies? It is
99. Which of the following is a characteristic of
the _________.
decision-making?

a. Buttocks
A. it only involves logical, rational thought
b. Head
B. it is often the result of many incremental
steps rather than one large step.
c. feet

C. It must always be done quickly in the health


d. hand
care setting.

2. To protect the vendor from the dangers of


D. they generally do not assume another
person’s point of view. rabies, PHN Cords advises him to clean the
wound thoroughly with soap and water, consult
100. Which of the following is true about a physician and receive anti-rabies vaccination.
effective creative thinkers? Which among the following vaccines can
provide active immunity?
A. They seldom generate new ideas and
alternatives.
1. Purified vero cell vaccine

B. They often say, “We’ve always done it this


way, so let us do it this way again.” 2. Human rabies immunoglobulin

C. They tend to analyze the components of a


3. Equine rabies immunoglobulin
problem.
4. Purified duck embryo vaccine d. Massive campaign for responsible pet
ownership

a. 1 and 4
Situation: The Field Health Services and
Information System (FHSIS) is recording and
b. 2 only
reporting system in public health care in the
c. 3 and 4 Philippines

d. 1 only

3. The vendor acquired rabies, what will PHN 5. The following are the objectives of the
Cords do to protect those who took care of FHSIS, EXCEPT:
him? He should administer____________.
a. Complete the picture of acute and chronic
disease
a. Pre-exposure prophylactic treatment only for
the family of Bert
b. Ensure data recorded are useful and
accurate and disseminated in a timely, easy to
b. Post-exposure prophylactic treatment
use fashion
only for the family of the vendor

c. Minimize recording and reporting burden


c. Pre-exposure prophylactic treatment to Bert
allowing more time for patient care and
and the vendor’s families
promotive activities

d. Post-exposure prophylactic treatment to Bert


d. Provides standardized facility-level data base
and the vendor’s families
which can be used for more in-depth studies

4. PHN Cord’s intervention to protect all


6. As a nurse, you should know the process of
residents who own pets, especially dogs,
how these information are processed and
should be done by_______.
consolidated. The fundamental block of the
FHSIS system is the ______________
a. Coordinating with city/ municipal agriculturist
for immunization of all pets
a. Family treatment record
b. Coordinating with city/municipal officials to
make an ordinance on stray dogs b. Output record

c. Massive campaign to families not to own pets c. reporting forms


at home
d. target/client list d. individual health record

7. The monthly field health service activity Situation: Nurse Omar takes care of the
report is a form used in which of the Ramos extended family who resides in the
components of the FHSIS? house owned by Nilda’s mother, Marta. Nilda
tends a variety store is married to Ramon, a
a. Target/client list government employee. They have four children:
Lester, 20 years old; Gina, 18 years old; Alex,
b. Output report 15 years old; and Celine, 12 years old. Lester is
a working student of his second year in
c. individual health record
computer technology course. Gina is a high

d. tally report school graduate; Alex is in third year high


school and Celine is in Grade six. There is one
8. In using the tally sheet, the recommended year old baby girl who is a daughter of Gina.
frequency in tallying activities and services Gina, however, could not pinpoint the one who
is_______. sired her child in as much as she had multiple
sex partners. This angered Ramon.
a. Weekly

b. Quarterly
10. Though he recognizes the remorse of his
c. Monthly daughter, Ramon still feels confused regarding
the situation. He said he tried his best to
d. Daily support his family and had always been
considerate and kind. He and his wife would
9. To monitor clients client registered in
always give them reminders and advice calmly
long-term regimen such as the Multi drug
and never in a nagging manner. But still they
Therapy, which component of the reporting
failed as parents. The possible nursing
system will be most useful?
diagnosis of Nurse Omar of his family
is_____________.
a. Output report

a. Interrupted family process


b. Tally report

b. Impaired parenting
c. target/client list
c. Parental role conflict ’s most practical and best advice would
be__________.
d. Ineffective role performance

a. Postpone sex and suggest other ways to


11. Nurse Omar ’s conversation with Gina
expressing love
revealed that the young woman still suffers from
a syndrome of failure: failure to complete one’s b. Explain to him the difference between sex
normal growth and development, failure to and love
complete education, failure to establish a
c. Teach him by step-to-step correct,
vocation and become independent and failure
continuous and consistent condom use
to have a life. The nurse Omar ’s intervention to
this problem is_____________. d. Discourage him on having a girlfriend and
focus more on his studies

a. Linear approach with regards to the


individual in the context of the family, 13. Ramon complains to Nurse Omar some
community and culture that will combat shame weird behaviour of Nilda. These past few
and guilt months, she has decreased sex drive, night’s
sweats and mood swings. He also received
b. Lay down the foundation of a future by
trusting human association and developing weird text message from her such as: “Do you
mutual trust initially with the nurse, then the really love me?”, “What role do I play in your
family, and eventually the whole community life?” “Do you still find me attractive?” The best
advice of Nurse Omar to Ramon should
c. Focus on the factors that will help protect
be________.
Gina towards proximal stimuli for healthy
growth and development to develop her
resiliency in confronting current and future a. Give her some money for make-over to
problem increase her self-esteem and make her look
attractive to him
d. Transform interactions among family
members, strengthen specific roles and b. Accompany her to a psychiatric
functions to strengthen family system in order to
eventually cope
c. Ignore his wife or tell her she is too old to act
like a teen-ager
12. In one home visit, Nurse Omar was
approached by the 15 year old Alex. He was d. Give reassurance that she is the best
asking about condom use. He said he has a person who came to his life
girlfriend with whom he is madly in love with but
does not want her to get pregnant. Nurse Omar
14. Apparently, the nurse interventions have a. “Well, your problem is easy to solve. I have
improved family relationships. The members here some cough syrup, ointments for the skin
are now communicating with one another and and some antibiotics. I will distribute this after
are excited in preparing for a family affair, which the meeting.”
is the baptism of Beatrice. Nurse Omar was
asked to be the godfather of the child. His best b. “Who among you here have children who are
response is_________. suffering from respiratory and skin diseases?
How about the adults who are here? Are you
a. Accept and proudly say that Beatrice will be also having the same problems?”
his 49th godchild
c. “I suppose you gave a lot of thought
b. Refuse and make an alibi that he belongs to about the problem and its possible solution.
another religion However, treating your children and the
elderly is not the first solution. We have to
c. Accept and express gratitude for the trust go to the root cause of the problem.”
accorded him by the family
d. “May I ask you what solution have you
d. Politely decline and explain that his identified for the community problem?”
relationship with the family must not go
beyond professional Situation: Nurse Lovely, a newly promoted
senior nurse in Obstetrics ward (OB) is
Situation: Belinda, the PHN in the Municipality attending a seminar on management and
of La Trinidad, learned from the residents that leadership in preparation for her work.
children and some elderly had been suffering
from respiratory and skin ailments allegedly due
to the bad smell curly dark smoke emitting from
16. Nurse Lovely learns the five principles of
the factory nearby. She was invited to the
goal setting in which the senior nurse must
community assembly that was initiated by the
provide enough time for OB nurse to improve
barangay council. performance. This is called__________

a. Challenge
15. The barangay captain asked, “What can
b. Commitment
you do to help solve the problem of Nurse
Belinda?” What would be the right response of
c. feedback
Nurse Belinda?
d. task complexity
17. The nurse also learns that continuous b. Demonstrates self-reliance in caring for
training is a personal as well as an herself
organizational goal. Choose the statements that
are true regarding continuous training c. Understands the communication of Nurse
Lovely regarding the services offered
1. Training employees is an excellent
investment and a cost to an institution d. Begins to have feeling belonging

19. Nurse Lovely took note that evaluating the


2. Continuous training is more of a personal
OB staff is an on-going function of
responsibility than institutional
management. Some of the reasons for
conducting evaluation include, EXCEPT to
3. Cross training and job rotation provide
_____________.
on-going part-time learning experience

a. Provide an indication of the costs of poor


4. Select the best people when hiring quality services
employees and invest their retention through
continuous training b. Justify the use of resources

c. Dissuade self-evaluation of OB staff


a. 3 and 4

d. Ensure that quality of care is provided by


b. 1 and 2
the OB staff

c. 1 and 4
Situation: You are an OB nurse in an
out-patient department of a hospital. You
d. 2 and 3
encounter pregnant women with complication
18. Noting the importance of
Nurse-Patient-Relationship, Nurse Lovely
reviewed Hildegard Peplau’s Theory which 20. A 35-year old woman, on her 2nd trimester
identified three phases, the FIRST of which is of pregnancy with insulin-dependent diabetes
when the pregnant woman is________. mellitus, comes to you for some advice. What is
the PRIORITY message for her at this time?
a. Feels the need to seek professional
assistance
a. Infants of diabetic mothers are big which can
result in more difficult delivery
b. Breastfeeding is highly recommended and performance was satisfactory. After a year
insulin use is not contraindicated though, she had to renew her PRC
registration and identification (ID) card.
c. Achievement of optimal glycemic control
is of utmost importance in preventing
congenital anomalies
22. What action must the nursing
d. Her insulin requirements will likely increase administration do FIRST?
beginning 3rd trimester of pregnancy

21. A 30-year old G6P5 woman at 12 weeks a. Report the matter to the head of office
who had the discretion to appoint the nurse
has just begun prenatal care. Her initial
laboratory reveals that she has human b. Verify with the Professional Regulation
immunodeficiency virus (HIV) infection. What Commission regarding the status of Nurse
would be a priority evidence-based nurse Dana

information for this patient?


c. Confront Nurse Dana and terminate her

a. Breastfeeding is still recommended due to d. Write a letter to the Civil Service Commission
the great benefits to the infant for proper action to Nurse Dana

b. Pregnancy is known to accelerate the course 23. It was found out that Nurse Dana did not
of HIV disease in the mother
pass the Nurse Licensure Examinations (NLE).
What legal action should be filled against her?
c. Medication for HIV infection is safe and
can greatly reduce transmission of HIV to
the infant a. Dishonesty

d. Breastfeeding will potentiate the transmission b. Conduct unbecoming of professional


of HIV from the mother to the child
c. Malpractice
Situation: Nurse Dana, a nursing staff
applicant, passed both written and oral d. Misrepresentation
examinations. Because she knows the head of
office, she promised to submit all her 24. In case Dana have medication error during
credentials after she has “fix things up.” She her tour of duty, the head of office can be liable
was appointed as Nurse I with a temporary because of the law called ____________.
status until she submits all her credentials,
including her PRC license. Her evaluation a. Unethical conduct
b. Respondeat superior b. “Nanay Isa, your intervention is entirely
wrong.”
c. Politicking
c. “It’s good you’re here. You can drive away
d. Res Ipsa Loquitur the spirits that entered the boy’s body.”

25. All Nurses must understand that after d. “You have to be sure that all the evil spirits
have been driven out of the boy’s body.”
graduation they should pass the NLE. To be
registered in the roster, they should take the
27. After a few minutes, Nanay Isa took a big
Professional Oath with a ________, EXCEPT.
bowl of soup and gave to the boy. The BEST
remark of the Nurse is _____________.
a. Member of Sangguniang Panlalawigan

a. “I also drink a soup when I get sick. How


b. Governor of Philippine Nurse Association
about you, Nanay Isa, do you do the same?”

c. Member of the Professional Regulatory


b. “The soup could have been better if you put
Board of Nursing
lemon grass on it.”

d. Provincial Governor
c. “Come on, tell me why soup must be given to
a child with fever
Situation: PHN Elfa works in barangay 14 and
15 in the Municipality of Agoho. One day, a d. “That’s correct. Increasing fluid intake will
neighbor summons her to attend to a 7-year old help lower down temperature.”
boy with high grade fever.
28. Finally, Nanay Isa took out from her pocket
a dried rose flower and place it on the boy’s
forehead. How will Nurse Elsa handle this
26. Upon reaching the house, a local action?
herbolaria, Nanay Isa was already attending to
the boy. She said that the boy played near the
a. Tell her not to use the dried flower again
river and the bad spirits entered his body. The because it does not have any good effect on the
MOST appropriate remark the nurse make is sick
_________________.
b. Ask for an extra piece of fried rose flower and
promise to use it
a. “Go on. Do what you have to do, then I
will take over.” c. Ask the herboloria the rationale for the
intervention
d. Leave the intervention as is. Anyway the a. Greenish yellow as differentiated from
intervention is neutral: not harmful nor mucoid white of trichomoniasis
beneficial
b. Mucoid white as compared to grayish-white
Situation: The local health board established a discharge of vaginosis
reproductive health clinic in the main health
center. Two nurses, Hunter and Irene, were c. Grayish white as differentiated from mucoid
white of chlamydia
assigned to handle services to address
problems related to sexuality, reproductive d. Yellowish white as compared to
health and fertility problems. trichomoniasis’ greenish-yellow

31. Nurse Hunter was invited by a women’s


group to give a lecture on healthy sexuality. In
29. Nurse Hunter classifies cases according to
the expectation check, he noted that there are
the major categories of reproductive tract
previous misconceptions expressed by the
infections. Which of the following is NOT part of
participants. Which of the following statements
such classification?
are correct?

a. Iatrogenic infections as aftermath of invasive


a. It is the obligation of the wife to give in to sex
procedures like catheterization and intra-uterine
every time he asks for it
device (IUD) insertion

b. Sexuality is fluid and may change


b. Urinary tract infections among male and
female patients
c. Effeminate men are gays

c. Sexually-transmitted infections
d. Homosexuality, being gay or lesbian, is an
abnormality
d. Endogenous infections resulting from poor
personal hygiene
32. One of the clients was positive to

30. Irene handles the screening for gonorrhoea Gonorrhea. Nurse Irene explained that

every two weeks among female sex workers in gonorrhoea and chlamydia, if left untreated can

the implementation of PD 856. In differential lead to Pelvic Inflammatory Disease (PID).

diagnosis of discharge among infected clients, Such condition may cause infertility due to

which of the following colors discharge will ______.

Irene take note to identify gonorrhoea from


other causes? gonorrhoea from other causes? a. Foul smelling odor discharge which can kill
the ovum
b. An unknown cause b. 7 days after the first day of the succeeding
menstrual
c. Scarring which can lead to tubal
occlusion c. 7 days before the first day of the succeeding
menstrual
d. Purulent discharge which can kill the sperm
d. 14 days before the first day of the
33. Nurse Irene further explained that a test succeeding menstrual

used to determine tubal patency using a


35. Ela insisted she might have been fertile
radiopaque material is the __________.
during the time of sexual intercourse. The nurse
explains that absolute period of fertility is the
a. Post-coital infetitlity test
span of time that a woman is likely to be

b. Sims Huhner test pregnant when she engages in unprotected


sex______.
c. Friedman’s test
a. Several days after ovulation
d. hysterosalpingography
b. During ovulation
Situation: Ela, 21 years old, is a law graduate.
She wants to review for the Bar but thinks she c. Immediately after ovulation
is pregnant. She said she has regular menses
but does not know when ovulation usually d. Immediately before ovulation

occurs. This have something to do her fertility


36. The nurse proceeded to take the menstrual
period during her last sexual intercourse with
history of Ela to find out if she is likely to be
her husband.
pregnant. Which of the following determines the
date of onset of last menstrual period (LPM)? It
is the ___________.
34. As a nurse, what would you tell Ela
regarding ovulation? The ovulation usually
a. Duration and character of the LMP
corresponds to the life of the corpus luteum
which occurs approximately _______________. b. Implantation bleeding

c. Spotting after the LMP


a. 14 days after the first day of the succeeding
menstrual
d. Bleeding before the last menstrual period
(LMP)
37. The nurse also asked about Ela’s B. January 20
secondary amenorrhea that would most likely
indicate her pregnancy. Secondary amenorrhea C. July 17
is cessation of menses for more than_______
months, after regular menstrual cycle has been
D. July 20
established.

40. The nurse noted the Fundic height of Anna


a. Five
is at the level of the umbilicus. In documenting
b. Three the data using Bartholomew’s rule, the most
probable age of gestation (AOG) in week is :
c. six

A. 12 weeks
d. four

B. 16 weeks
38. The nurse also asked for presence of
secondary dysmenorrhea. Which of the C. 20 weeks
following conditions is not INCLUDED under
secondary dysmenorrhea? D. 32 weeks

Situation: A pregnant woman is monitored


a. Intra-uterine device (IUD)
continuously for a range of signs and symptoms
b. Pelvic Inflammatory Disease (PID) that indicate potential complication. Nurse Carol
attends to different discomforts and problems of
c. Malposition of the cervix pregnant women.

d. Absence of any underlying anatomic


abnormality
41. Alice an O.P.D nurse admitted Mrs. Felia to
39. Carla another patient in the clinic just found
the antepartum unit with a diagnosis of severe
out that she is pregnant. She asks when would
Hyperemesis Gravidarum. When the nurse
be her delivery date. What is the expected date
reviews the laboratory tests, she would expect
of confinement (EDC) of a pregnant woman
which of these findings?
whose menstruation was from April 10 to April
13?
A. Increased hematocrit

A. January 17 B. Decreased blood urea nitrogen


C. Increased potassium acceleration lasted for 20 seconds and
occurred 3 times during the 20 minute test. The
D. Low urine specific gravity Nurse is correct in interpreting the test as a:

42. Nurse Alice suspects presence of sexually


A. Reactive test
transmitted infection to Mrs. Felia specifically
Syphilis to a pregnant client. Which of the
B. Non- reactive test
following tests will be recommended to the
client to confirm diagnosis? C. Positive test

A. Complete blood count D. Negative test

B. Urinalysis 45. Another client is also scheduled for


Amniocentesis. Nurse Carol explains to the
C. Benedict’s test client that one of the risks of amniocentesis is:

D. VDRL
A. Rupture of membranes

43. Mrs. Ilagan a 26 year old Primigravida is


B. Premature labor
being prepared for a nonstress test. This is an
assessment test based on what phenomenon? C. Fetal death

A. Braxton-Hicks contractions cause fetal heart D. Malformation


rate alterations.
46. The nurse is assessing the fetal heart
B. Fetal heart rate slows in response to a monitor strip of a client having a contraction
uterine contraction. stress test. Which of the following, if noted by
the nurse, would indicate a negative test?
C. Fetal movement causes an increase in
maternal heart rate.
A. No late decelerations after any
D. Fetal heart sounds increase in connection contractions on a strip with three
with fetal movement. contractions within a 10-minute time frame.

44. After a non-stress test is completed, Nurse B. Late decelerations after at least 2
contractions on a strip with three contractions
Ilagan observes on a monitor of the fetal strip
within a 10-minute time frame.
results that the fetal heart rate accelerated 15
BPM with each fetal movement. The
C. Late decelerations after one contraction on a B. The baby's head recedes upward between
strip with three contractions within a 10-minute pushing contractions.
time frame.
C. The perineum is thin and stretching
D. An increase in fetal heart rate after three around the occiput.
contractions within a 10-minute time frame.
D. The mouth and nose are being suctioned.
47. The client complains of feeling tired and
thirsty. The nurse evaluates that the mother 49. To deliver her infant, a woman is ask to
understand the reason for taking only small sips push with contractions to deliver. Ensuring the
of water and ice chips during labor. Which of standards of nursing practice, which of the
the following statement expressed by the following is the most effective and safest
mother would reflect she understands the pushing technique to teach her?
situation? When:
A. Lying supine with legs in lithotomy stirrups.
A. the body normally has a sufficient store of
energy B. Squatting while holding her breath.

B. the digestive process is normally slower C. Head elevated, grasping knees, breathing
during labor out.

C. the intestinal tract should be completely D. Lying on side, arms grasped on abdomen.
empty before delivery in order to avoid infecting
the baby 50. The delivery room nurse based on the
standards of nursing practice, episiotomy is
D. Cesarian section is always a possibility even
usually indicated for which of the following
in normal labor
purposes?
48. Kim, a 27 year old, multigravida client, has
been transferred to the delivery room after A. To prevents distention of the bladder.
spontaneous rupture of membrane and
B. To relieve pressure on the fetal head.
crowning was noted by the nurse in charge. You
know that your teaching has been effective
C. To aid in contraction of the uterus following
when the laboring client's partner shouts, "She's delivery.
crowning!" as:
D. Done primarily for the physician’s benefit.

A. You first start to see a little of the baby's


head.
Situation: It is important for an Obstetric Nurse 53. The delivery room nurse palpates the
to perform a comprehensive physical client’s fundus immediately after delivery of the
placenta and assess that it is boggy. The nurse
assessment after labor and delivery that could
massages the patient’s uterus until it is firm.
predispose the mothers to potential Considering evidence-based nursing practice,
complications such as hemorrhage. which medication would the nurse anticipate
might need to be administered if the uterus
becomes boggy again?

51. The Nurse in the delivery room is attending A. Oxytocin (Pitocin)


to Mrs. Cruz on labor to make sure that
B. Ibuprofen
maternal injury will be prevented during the
postpartum period. Which of the following
C. Rho (D) immnune globulin (RhoGAM)
instruction should the nurse consider to prevent
postpartum hemorrhage? D. Magnesium sulfate

54. Mrs. Evita 28 years old gave birth through


A. Massage the fundus regularly
Cesarian section. The Nurse examines her and
B. Postpone breastfeeding of the baby- identify the presence of lochia serosa and feels
the fundus 4 fingerbreadths below the
C. Apply warm compress to her abdomen umbilicus. This indicated that the time elapsed
is:
D. Have bed rest and avoid early ambulation

52. When the placenta has been delivered, the A. 1 to 3 days postpartum
first thing the nurse should do in adherence with
B. 4 to 5 days postpartum
the standards of nursing practice is to:

C. 6 to 7 days postpartum
A. Inspect the placenta for completeness of
the cotyledons D. 8 to 9 days postpartum

B. Palpate the uterus to see if it is contracted 55. In assessing a new mother’s response to
her son’s birth on the first post partum day,
C. Administer oxytoxic agents as ordered
which behavior does the Nurse expect to find

D. Estimate the blood loss to detect any present?


bleeding
a. Talkativeness and dependency
b. Autonomy and Independence B. May resume with regular exercise if minimal
bleeding has been noted.
c. Disinterest in her own body function
C. Avoid sexual intercourse.
d. Interest in learning to care for the baby
D. Avoid intake of spicy foods
Situation: A Maternal-Child staff nurse is
attending to the pregnant mothers with varied 58. Another pregnant mother wants to be

obstetric disorers. A comprehensive clarified on her laboratory studies which reveals

assessment was conducted. One of the clients blood Type -A and she is Rh negative.

seeks further question regarding Placenta Problems related to incompatibility may develop
in her infant if the infant is:
Previa.

A. Type O

56. Which of the following would be the B. Rh positive

physiologic basis for a Placenta Previa?.


C. Delivered preterm

A. A loose placental implantation. D. Type B, Rh negative

B. Low placental implantation. 59. An Obstetric nurse is assessing a 39 year


old pregnant woman who is married to an
C. A placenta with multiple lobes.
American citizen and Rh negative, is seen by
the Physician during the first trimester of
D. A uterus with a midseptum.
pregnancy. A test to detect presence of
57. A patient diagnosed with Placenta Previa antibodies was conducted to her. The nurse’s
should be given specific instruction before teaching is effective if the client understands
discharge from the hospital. To ensure that she will first receive Rho (D)
standards of nursing practice, which among the immunoglobulin (RhIg):
following should be considered by the nurse as
part of instruction to the client? A. If the result of Indirect Coomb’s test is
positive

A. Eat a low calorie diet


B. If the result of Indirect Coomb’s test is
negative
C. If the result of Direct Coomb’s test is positive B. ask her to push with the next contraction so
delivery is rapid.
B. If the result of Direct Coomb’s test is
negative C. assess blood pressure and pulse to detect
placental bleeding.
60. During the prenatal visit the Nurse explains
further to a client who is Rh negative that D. attach a fetal monitor to determine fetal
status.
RhogGAM will be administered:

62. Nurse Jamie in a labor room is preparing to


a. Weekly during the ninth month, because this care for a hypertonic uterine dysfunction. The
is her third pregnancy
nurse observed that the client is experiencing
uncoordinated contractions and erratic in their
b. Within 72 hours after delivery if infant is
found to be Rh positive frequency, duration and intensity. The priority
nursing intervention in caring for the client is to:
c. During the second trimester , if an
Amniocentesis indicates a problem
A. Monitor the oxytocin (Pitocin) infusion closely

d. To her infant , immediately after delivery if the


Coomb’s test is positive B. Provide pain relief measures

Situation: The birth process affects the C. Prepare client for amniotomy

physiologic systems of the mother and the


D. Promote ambulation every 30 minutes
fetus. Staff nurse Jamie is assigned in the
Labor and Delivery Room Area. 63. Mrs. Barbara 34 years old is being
admitted in the hospital unit for severe
Preeclampsia. When deciding on where to

61. A G7P 6 woman is in the hospital only 15 place her, which of the following areas would be

minutes when she begins to deliver most appropriate?

precipitously. The fetal head begins to deliver


as you walk into the labor room. The best action A. By the nursery so she can maintain hope she
will have a child.
of Nurse Jamie would be to:

B. Near the elevator so she can be transported


A. place a hand gently on the fetal head to quickly.
guide delivery.
C. Near the nurse’s station so she can be
observed closely.
D. In the back hallway where there is a quiet, 66. The best way for an infant’s father to help
private room. his child complete the developmental task of
the first year is to:
64. The Physician orders intravenous
Magnesium Sulfate for Mrs. Barbara. Which of A. expose her to many caregivers to help her
learn variability.
the following medications would the Nurse has
readily available at the client’s bedside?
B. talk to her at a special time each day.

a. Diazepam (Valium ) C. respond to her consistently.

b. Calcium Gluconate D. keep her stimulated with many toys.

c. Hydralzine (Apresoline ) 67. Whenever the parents of a 10-month-old


leave their hospitalized child for short periods,
d. Phynetoin ( Dilantin )
he begins to cry and scream. The nurse
explains that this behavior demonstrates that
65. Which of the following signs would alert the
the child:
Nurse that Mrs. Barbara’s whose latest blood
pressure 160/110, may be about to experience
A. Needs to remain with his parents at all times.
a seizure?

B. Is experiencing separation anxiety.


a. Decreased contraction intensity
C. Is experiencing discomfort.
b. Epigastric pain
D. Is extremely spoiled.
c. Decreases temperature
68. Dennis, a preschooler sees you pour his
d. Hyporeflexia
liquid medicine from a tall, thin glass into a
short, wide one, he will probably reason that:
Situation: As a Pediatric Nurse you are
confronted with varied concerns regarding
A. the amount of medicine is less (the glass
growth and development from parents,
is not as full).
teachers, and children. The nurse recognizes
the need for health supervision and anticipatory B. the amount of medicine did not change, only
guidance for these groups. the appearance.
C. pouring medicine hurts it in some way time to attend to their needs.
because it changes.

D. the glass changed shape to accommodate


the medicine. 71. Elvis 8 months old was diagnosed with
Acute Laryngotracheobronchitis (LTB) and is
69. A school nurse prepares a lecture on
managed inside a mist tent. As Nurse Paterno
Puberty changes for first year high school girls.
conducts assessment, which of the following
She asks the group, "What is the first sign of
observations would lead her to suspect that
Puberty?" A student correctly replies :
airway occlusion is occurring?

A. "The appearance of breast buds."


A. He states he is tired and wants to sleep.

B. "An increase in energy and appetite."


B. His respiratory rate is gradually
increasing.
C. "The occurrence of the first menarche."
C. His cough is becoming harsher.
D. "Appearance of body odor."
D. His nasal discharge is increasing.
70. When encouraging the hospitalized
physically challenged or chronically ill 72. A child is scheduled for a Myringotomy with
adolescent to develop and maintain a sense of placement of Tympanostomy tube. What is the
identity, you would: goal of this procedure that Nurse Paterno will
discuss with the parents?
A. provide the opportunity for individual
decision making.
A. To decrease infection in the ear

B. provide physical comfort to the individual.


B. To irrigate the eustachian tube

C. ask the parents what the adolescent is


C. To correct a malformation in the inner ear
capable of doing.

D. To equalize pressure in the tympanic


D. provide care until the adolescent insists on
membrane
being independent

73. An 8 year old female child was admitted in


Situation: Nurse Paterno a Pediatric Nurse
the hospital with medical diagnosis of Acute
enjoys taking care of children in the ward even
Rheumatic fever. When obtaining a health
though it is so difficult and takes so much
history from the child’s mother, the nurse should D. Heart failure
ask the questions to determine if the child was
recently ill with: Situation: There are varied Pediatric disorders
that require comprehensive assessment and
nursing interventions. The following scenarios
A. Mumps refer to health problems of children.

B. Measles

C. a viral flu 76. A 5-week-old infant is brought to the


pediatrician’s office with symptoms of irritability,
D. a sore throat
weight loss, and projectile vomiting. On physical
examination, the infant appears dehydrated.
74. You would teach the mother of a boy with
From these symptoms, you know that the infant
Tetralogy of Fallot (TOF) that if he suddenly
probably has:
becomes cyanotic and dyspneic to:

A. Hirschsprung’s disease
A. place him in a semi-Fowler’s position in an
infant seat.
B. Tracheoesophageal Fistula
B. lie him supine with the head turned to one
side. C. Pyloric stenosis

C. lie him prone, being sure he can breathe D. Intussusception


easily.
77. Pediatric Nurse admitted a post cleft
D. place him in a knee-chest position. palate repair child and immediately the nurse
should position the child:
75. Dyspnea, cough, weight gain, weakness,
and edema are classic signs and symptoms of
A. Left side lying.
which condition?
B. Prone.
A. Pericarditis
C. Dorsal recumbent.
B. Hypertension
D. Semi Fowler's.
C. Myocardial infarction (MI)
78. Another neonate is suspected of having a C. All the pants have become tight around
tracheoesophageal fistula. Priority nursing care the waist.
until the diagnosis is confirmed includes:
D. The child prefers some salty foods more than
A. monitoring the neonate carefully during and others.
after feedings
Situation: A newly married couple Berta and
B. elevating the neonate’s head after feedings Bart wants to practice Family Planning to
prepare a good future for their family. Nurse
C. feeding only glucose
Bing a Family Planning Counselor is planning a

D. feeding nothing by mouth lecture regarding the different methods of family


planning.
79. Upon interviewing the parents of the child
with Acute Glomerulonephritis, the nurse
understands that which information collected is
81. Which of the following family planning
most often associated with this condition?
method which identifies the fertile and infertile
days of the menstrual cycle as determined
A. Nausea and vomiting for the last 24 hours
through a combination of observations made on
the cervical mucus, basal body temp recording
B. Streptococcal throat infection 2 weeks
prior to diagnosis and other signs of ovulation?

C. History of urinary tract infection for 5 days A. Basal Body Temperature

D. Pruritus for 1 week prior to diagnosis B. Standard Days Method

80. A newly admitted 5-year old child in the C. Sympto-thermal Method


Pediatric ward is diagnosed with Wilm’s Tumor.
Upon initial interview, the nurse would be most D. Lactational Amenorrhea Method
concerned about which statement by the child’s
82. During a family planning seminar
mother?
conducted in the Barangay Health Center,
Nurse Bing was asked by Berta, a married
A. My child has lost 3 pounds in the last month.
woman who wants to try using contraceptives, if
B. Urinary output seemed to be less over the it is true that contraceptives will render couples
past 2 days. sterile. Nurse Bing’s response should be:
A. “Yes, It’s true.” C. “Yes, family planning can cause abortion.”

B. “Yes, If you are already using it for more than D. “Yes, if the couple is using the artificial
3 months.” methods of family planning.”

C. “No, It will not cause sterility if you are also 85. Mackle-More, a Public Health Nurse (PHN),
using condoms.” is assigned in conducting seminars on Family
Planning Program in the different Barangays.
D. “No, Once you stop using the
contraceptive method, you can have She is aware that the roles of PHNs on Family
children again.” Planning Program are the following EXCEPT:

83. Bart the husband, further asked Nurse Bing


A. Provide counseling among the clients to help
if contraceptive method will result to loss of increase family planning acceptors and avoid
sexual desire. Nurse Bing’s most appropriate defaulters.
response would be:
B. Ensure availability of family planning
supplies and logistics for the PHNs and
A. “No, but it will make you uncomfortable with other barangay health workers only.
your sexual relationship.”
C. Provide packages of health services among
B. “Yes, it causes lack of sexual desire of the reproductive age group in all health facilities.
male partner.”
D. Inform the clients about the importance and
C. “Yes, it causes lack of sexual desire of the benefits/advantages/disadvantages of family
female partner.’ planning.

D. “No, it can actually enhance your sexual Situation: Nurse Maja continues to expand her
relationship.” roles by actively participating in the activities of
the Health Center in collaboration with the
84. In a CHN class, a student asked Mr. Pablo,
Department of Health.
the Clinical Instructor, if family planning
methods can cause abortion. As an instructor,
Mr. Pablo’s response should be:
86. Which of the following is not included in the

A. “No, family planning prevents pregnancy, Child Health Programs of the DOH?
but it does not terminate pregnancy."
A. Adolescent Screening
B. “No, family planning puts a pregnant woman
at risk for miscarriage, but not abortion.”
B. Expanded Program on Immunization D. giving the baby breast milk only. Drops or
syrups consisting of vitamins, mineral
C. Dental Health supplements, or medicines should not yet
be given until the 6th month of life.
D. Micronutrient Supplementation
89. The following are the benefits of
87. Breastfeeding is the most essential feeding breastfeeding to the infants EXCEPT:
for infants that has nutritional , immunologic
values and maternal advantages for the mother. A. Provides a nutritional complete food for the
Exclusive breastfeeding during the first young infant.

half-year of life is an important factor that can


B. Strengthens the infant’s immune system,
prevent: preventing many infections.

A. Infant and childhood morbidity and C. Safely dehydrates and provides essential
mortality nutrients to a sick child.

B. Infant and childhood Mental Disorders D. Increases IQ points.

C. Occurrence of Cancer 90. During a Ward class in the Obstetric Ward


of a community hospital, a mother asked the
D. Occurrence of Heart Disease Nurse regarding the benefits of breastfeeding to
the mothers. The Nurse best response would
88. Rona, G1P1, is on her 2nd post partum
be:
day. She asks Nurse Maja about the definition
of exclusive breastfeeding. Nurse Maja
A. “It increases the woman’s risk of excessive
responds based on his knowledge that blood loss after birth.”
exclusive breastfeeding means:
B. “It reduces the woman’s risk of excessive
blood loss after birth.”
A. giving the baby breast milk and water only.

C. “It provides artificial methods of delaying


B. giving the baby breast milk and solid food
pregnancies.”
only.

D. “It increases the risk of ovarian and breast


C. giving the baby breast milk and drops or
cancers and osteoporosis.”
syrups consisting of vitamins, mineral
supplements, or medicines only.
91. Nurse Dorothy is preparing to administer B. immunity from Measles is lifelong after
vaccinations to children. She knows that the the first attack.
following are correct EXCEPT:
C. no immunity is induced by the infection.
A. The vaccination schedule should not be
restarted from the beginning even if the interval D. immunity from Measles is just for 6 months
between doses exceeded the recommended after the first attack.
interval by months or years.
94. When can you say that a child is already a
B. Giving doses of a vaccine at less than the
“Fully Immunized Child”?
recommended 4 weeks interval may lessen the
antibody response.
A. If he received one dose of BCG, 3 doses of
C. Lengthening the interval between doses of OPV, 2 doses of DPT, 3 doses of HB and one
vaccines leads to higher antibody levels. dose of measles before his/her first birthday.

D. Use one syringe one needle for all the B. If he received two doses of BCG, 3 doses of
children receiving the same vaccination. OPV, 3 doses of DPT, 3 doses of HB and one
dose of measles before his/her first birthday.
92. Rose, a mother of a 7-month-old baby, is
asking the nurse in the Health Center regarding C. If he received one dose of BCG, 2 doses of
OPV, 3 doses of DPT, 3 doses of HB and one
the 7 vaccine preventable diseases. All of the
dose of measles before his/her first birthday.
following diseases are included EXCEPT:
D. If he received one dose of BCG, 3 doses
A. Diphtheria of OPV, 3 doses of DPT, 3 doses of HB and
one dose of measles before his/her first
birthday.
B. Measles

95. Annabelle, a new mother, asks the nurse


C. Poliomyelitis
about the purpose of the first BCG vaccination

D. Dengue given to her son. The nurse’s best response


should be:
93. Kurt, 4 years old, has Measles. His mother
asks the nurse if there is a chance that Kurt will A. “An early start with BCG reduces the chance
contract the virus again. The nurse’s response of severe pertussis.”
should be based on her knowledge that:
B. “The extent of protection against polio is
increased the earlier the BCG is given.”
A. reactivation of old infection is common with
Measles.
C. “BCG given at earliest possible age D. Anti-gout
protects the possibility of TB meningitis and
other TB infections in which infants are 98. Ulasimang Bato or Pansit-pansitan is used
prone.”
to:

D. “An early start of BCG reduces the chance of


being infected and becoming a carrier. It A. lower cholesterol levels
prevents liver cirrhosis and liver cancer.”
B. lower blood sugar levels
Situation: Barangay Dionisia is situated in a
remote area. The Public Health Nurse C. lower ammonia levels
conducted several health training programs
D. lower uric acid levels
regarding Herbal plants that would be useful in
the treatment of illness and health problems.
99. This refers to a drug outlet managed by a
The following can be found in the small garden
legitimate community organization,
of the Barangay.
non-government organization, and the local
government unit with a trained operator and a
supervising pharmacist, and specifically
96. Tsaang Gubat is used to treat which of the licensed by the Bureau of Food and Drugs to
following? sell, distribute, offer for sale, and or make
available low-priced generic home remedies,
A. Diarrhea and Stomachache Over the Counter (OTC) drugs, antibiotics, and
medication for chronic diseases.
B. Cough and Fever

A. Mercury drugs
C. Colds and Pain

B. Right Med
D. Hypertension

C. Generic Pharmacy
97. Niyug-niyogan is an:

D. Botika ng Barangay
A. Analgesic
100. One strategy to address the problem in a
B. Anti-helminthic poor Barangay aside from Herbal Plants is food
production. Which of the following is a priority?
C. Anti-hypertensive
A. A community managed poultry and piggery d. Request the nurse supervisor to give her
brief orientation before compliance
B. Planting plenty of Malunggay
2. To qualify as an operating room nurse in the
C. Planting tomatoes and eggplants in Philippines setting, Nurse Luna should possess
containers the minimum requirements of _____________.

D. Engaging in a home –based food processing


business a. Master’s degree holder with valid and current
license

RECALLS 7 - NP3 b. Worked in the surgical unit for 8 hours

Situation: Nurse Luna is employed in hospital c. RN and has worked abroad


“X” and assigned in the Medical Ward for a year
now. The nurse supervisor ordered her to d. RN with valid and current license and
Surgical Ward orientation
proceed immediately to the Surgical Ward as a
reliever to another nurse who went on 3. Nurse Luna is a graduate in the Philippines
emergency sick leave. She was not oriented in nursing school. As part her professional and
the Surgical Ward and the unit was very busy. personal development, she should attend which
of the following program?
EXCEPT___________
1. Nurse Luna is employed in hospital “X” and
assigned in the Medical Ward for a year now. a. Programs by the Philippine Nurses
The nurse supervisor ordered her to proceed Association

immediately to the Surgical Ward as a reliever


b. Continuing Professional Development
to another nurse who went on emergency sick
programs by the Professional Regulation
leave. She was not oriented in the Surgical Commission
Ward and the unit was very busy.
c. Symposium and forum offered by the
school
a. Request the nurse supervisor to assign a
more experienced nurse reliever
d. Programs of international nurses
associations
b. Refuse the order of the nurse supervisor and
stay put in the medical ward
4. The PRIORITY objective behind career
advancement of Nurse Luna is ____________.
c. Comply with the order of the nurse supervisor
a. Increasing revenue of the service providers b. Avoid contact with fur-bearing pets

b. Renew old acquaintances and establish c. Avoid going to malls


camaraderie
d. Wash bed sheets in warm water
c. Increased number of networking activities
7. Mr Gilbert is for postural drainage. The
d. Updating one’s knowledge, skills, conduct nurse should position the client’s head at
and values in professional nursing
______________.

5. Nurse Luna has an expired license but


promises to renew her license in due time. a. No greater than a 25 degree downward
angle
Which of the following violation can she be
charged if she participated in home health care
b. A 30 degree lateral angle for 25 minutes
activity?
c. 25 degree at lateral angle
a. Malpractice
d. A 30 degree downward angle for 25 minutes
b. Grave coercion
8. Nurse Beth is teaching Michel, an
c. Felony asthmatic, on how to use the Spirometer. She
should instruct the client to have the
d. Negligence mouthpiece________________.

Situation: Health Education is an area of


a. Place into the mouth and have regular
nursing practice when the nurse can be creative
breathing
and independent in the work setting. The
following questions apply. b. Place into the mouth and have a fast deep
breath

c. Place into the mouth and inhale slowly


6. A nurse is developing a Teaching plan for
Isabel 18 year old with Bronchial Asthma. She d. Place into the mouth and exhale slowly
has an order for discharge. Which part of the
8. Nurse Beth is teaching a client on how to use
teaching plan should be given PRIORITY?
metered dose inhaler to prevent asthmatic
attack while in the hospital. She should instruct
a. Quick relief medicines as ordered
the client to do the following 11. Which of the following questions should you
__________________ EXCEPT. ask during an admission interview for a client
with a diagnosis of pheochromocytoma?
a. Keep the head of the bed at 15 degree
angle a. Do you always feel like you are suffocating,
you want to rest and sleep
b. Do oral care after use of the inhaler
b. Do you suddenly feel warm and flushed
c. Use the inhaler before she take her meals when you get out of bed

d. Use the inhaler as ordered c. Do you notice an increase in your heart


beat?
10. You are conducting health-teaching
sessions to clients with cardiovascular d. Do you have an increase in urination lately?

disorders. Client Pedro asks you this question:


12. When the sympathetic nervous system is
“Tell me, Nurse, what I should do with my
stimulated in the case of pheochromocytoma,
Hypertension?” The best response of a Nurse
you expect which of the following signs?
is____________.

i. Hypertension
a. “comply with your diet, lifestyle and exercise”

b. “strictly follow your prescribed daily exercise ii. Headache


and smoking cessation”

iii. Hyperhidrosis
c. “comply with your diet, life style
modification and prescribed medicines”
iv. Hypermetabolism
d. “include garlic in your meals with regulation
of alcohol consumption”
a. 3 and 4

Situation: As a staff nurse in a government


b. 1 only
hospital, you have been exposed to varied
cases of clients with endocrine problems. Your c. 1, 2, 3, and 4
nursing responsibility starts from admission to
discharge which is a domain of your d. 2 and 3
competencies.
13. Which of the following drugs can induce
hypertensive crisis in Pheochromocytoma?
a. Tricyclic antidepressant Situation: Mr. Con is being prepared for a
major surgery. Legal preparation for surgery
b. Corticosteroid consists of checking all the required forms for
the operation. Equally important is to make sure
c. Respiratory stimulant
that the patient is physically, psychologically,
d. Radio iodine therapy and emotionally ready for the procedure.

14. In the presence of pheochromocytoma, the


diagnostic test which is expected to be elevated
16. Informed consent is a process that gives
is ___________.
the patient opportunity involved in his or her
care. As patient advocate, the nurse ensures
a. Serum thyroid hormone levels
the following three conditions are present to
make consent valid, EXCEPT:
b. Albumin globulin test

c. Urine cyclic adenosine mono phosphate a. Adequate disclosure of the diagnosis by the
physician
d. 24 hours urine collection for
vanillylmandelic acid (VMA) b. Comprehension of information by the patient
before the operation
15. Palpation, as a modality for physical
examination is AVOIDED when diagnosed with c. Patient voluntarily giving consent

pheochromocytoma because this


d. Forms signed by any close relative or
action_______________.
watcher

a. Will cause sudden release of norepinephrine 17. The patient asks you, “What do you think of
and severe hypotension my surgeon?” You answered “hmmmmm… he
is not really the best one and he seems not to
b. Will cause a sudden release of
care for patient...” As a result, the patient
cathecolamines and severe hypertension
switches to another surgeon. The latter may
c. Will displace the location of the tumor have grounds to sue you for _____________.

d. Will cause sudden release of epinephrine


a. Slander
and severe palpitation

b. Invasion of privacy
c. Malpractice Situation: Conducting Research is one of the
major roles of the nurses both in hospital and
d. Libel community settings. To be able to develop such
competencies, the nurse has to undergo an
18. One of your patient’s visitors whisper to
actual conduct of the research process.
you, “I hope you will not try to revive my dear
friend if her heart stops as she has already
suffered a lot.” The correct response is
___________. 20. Which of the following statements BEST
described a researchable problem?

a. “That decision is up to the physician”


a. Responses of parents toward having children
b. “We are all trained in cardiopulmonary with congenital heart diseases.
resuscitation”
b. The relationship between relaxation
c. “There is a ‘ Do not resuscitate’ order in her technique and relief of pain of post CABG
chart” patients in the surgical coronary care unit.

d. “ I understand your concern, but I can’t c. Incidence of medication errors and reporting
discuss this matter with you” practices of Health Care Professional in a
teaching hospital
19. According to the Joint Commission, the
most frequently cited factor in sentinel d. To what extent do pre-operative teaching
affect the length of hospitalization of
(unanticipated) events that leads to a patient’s patients going for surgery
serious physical or psychological injury is
______________________. 21. Nurse Joan has to undergo literature
search for her study. She can avail of this from
a. Confusion within the health team the following EXCEPT_____________.

b. Miscommunication among health team a. A summary of research articles that are


members relevant to the study

c. Incompetence by a team member b. A written document published by the


investigator herself
d. Policy changes are not followed by adequate
and consistent staff education c. Any retrieval from website that will help her
search for the subject on investigation
d. A description of the scientific study from a. Retrospective evaluation
an information provided by a faculty
member teaching research b. Field setting for the study

22. Weight is taken as a baseline c. Comparable group


measurement of obese female adolescents as
study subjects for a weight reduction program. d. Manipulation of the dependent variable
This is repeated to note any changes. This
Situation: Kian, 16 year old, a foreigner was
pre-test is done to___________________.
admitted in the medical ward due to abdominal
pain, nausea and vomiting by Nurse Tessie.
a. Determine whether the instrument is
defective

b. Assess if research design is appropriate to


25. In initiating care for patient Kian, which of
the problem identified
the following would be an APPROPRIATE
c. Evaluate whether the instrument is defective question to be asked by Nurse Tessie in her
assessment?
d. Obtain preliminary data before a
treatment is conducted by the researcher
a. Since this is doctor’s order, you have to drink
ice water, instead of hot tea.
23. A Nurse researcher is using ACCU-CHEK
a monitoring kit to test presence of Diabetes b. Do you have any books I could read about
Mellitus among her study subject. How do you people of your culture?
classify this type of measurement?
c. Do you need to set aside your cultural
practices, and comply with hospital rules and
a. Microbial regulations?

b. Cytological d. Is there anything I am doing that is not


acceptable to your culture?
c. Physiological
26. Nurse Tessie respects cultural practices
d. Chemical integration in her nursing care plan. Which of
the following nursing action is MOST
24. Nurse Joan, wanted to conduct a study
represented of the culturally competent nurse?
using quasi-experimental design. This design
will need a __________.
a. Help patient Kian to learn and understand the b. Demonstrate people being responsible for
language their life patterns

c. Is non-denominated community service


b. Explain and validate health knowledge and
beliefs of Patient Kian with that of the hospital
d. Formalizes a religious dogma

c. Help Patient Kian identify ways to relate more


29. Positive Practice Environment (PPE)
to the culture where they now resides
influences healing process. Which of the
d. Ask patient Kian to help Nurse Tessie in following ways can help Nurse Tessie create a
knowing more the culture of his origin healing environment?

27. The family of Patient Kian request


a. Ensure that relatives and friends visit the
utilization of warm compress with banana patient
leaves to Patient Kian. Which of the following is
the MOST appropriate response of Nurse b. Empower clients to make healthy decisions
Tessie? Alternative therapies_________. for themselves

c. Place television in each room of the hospital


a. Cost less than traditional therapies
d. Ensure that staff nurses does not
b. Are use when traditional therapies are not experience burnout
effective
Situation: Julie, 28 years old, has been
c. Utilized natural products while traditional
diagnosed with Diabetes Mellitus. She was
therapies do not
advised by her family physician to be admitted
d. Can be effective as traditional therapies to undergo preservation for insulin therapy. Her
for some conditions blood sugar ranges from 200 to 210 mg/dL. At 6
am, Nurse Cynthia administered her insulin
28. Patient Kian’s family requests time for
injection. After 2 hours, the patient complained
spiritual healing process in the hospital. This is
of cold clammy perspiration, chilly sensation
allowed by Nurse Tessie and hospital because
and abdominal discomfort.
it______________________.

a. Gives fulfillment and meaning to the


patient and family 30.Which of the following PRIORITY nursing
actions should the nurse perform?
a. Give her biscuit to eat c. Let the attending physician decide on the
necessity of the treatment

b. Do urine testing for sugar


d. Respect the decision of the client

c. Provide her warm blanket 33. You are taking care of Mr Dencio who is on
the last cycle of radiation therapy for his lung
d. Take blood pressure and put her on bed rest cancer. You should instruct Mr Dencio to

31. Patient Julia has been classified to


a. Brush teeth and gums vigorously after meals
have a type II Diabetes Mellitus. Which of the
following is NOT a typical manifestation of
b. Wait one hour after treatment before eating
individuals with this condition?
c. Use mouthwash containing alcohol every 2
a. Frequency of urination hours

b. Increased craving for food d. Avoid drinking hot fluids

c. Increased thirst Situation: Ime is the Nurse on duty in the


medical ward and many of her patients are
d. Weight loss suffering from problems of oxygenation.

Situation: Mr Dencio, 58 years old is admitted


to the pay ward because of respiratory problem.
34. The following are relevant data to be
The nurse initiated oxygen treatment by mask
documented when taking the health history of
but the client refuses despite the
a client with anemia EXCEPT:
encouragement by the wife. The client is aware
of the benefits of the treatment.
a. Alcohol intake

b. Fatigue and weakness


32. Which of the following should be given
priority? c. Dietary intake

d. Episodes of bleeding
a. Ask the opinion of the wife

35. A client with congenital heart disease is


b. Conduct consensus building
suffering from thickening of the skin under his
fingers due to chronic hemoglobin desaturation. d. Cough
Which of the following specific term should Ime
use to accurately describe MOST the client’s 38. A client is on a diuretic therapy. Expected

condition in the chart? entry in patient’s chart should include the


following information, EXCEPT:

a. Peripheral cyanosis
a. Serum electrolytes monitored
b. Pallor of the finger tips
b. Intake and output recorded
c. Peripheral neuropathy
c. Lasix administered at 8 o’clock in the
d. Clubbing of the fingers evening

36. When the Nurse is assessing a client with d. Weight is taken before drug is given

Congestive Heart failure with pitting edema, the


Situation: Maya, a 42 year old teacher with
Nurse’s documentation will include which of the
cardiac ailment, nervously informs the doctor
following:
that her goiter is getting bigger and distracts her
while swallowing food. The physician who
a. Degree of pitting edema
examined her instructed the nurse to admit
Maya and to prepare her for surgery after
b. Time of indention recovery
medical clearance.
c. Depth of edema

d. All of the options


39. While interviewing Patient Maya, she claims
37. Mr. Gabby is with left sided heart failure. that she is anxious for the coming surgery. You
Ime’s documentation of her assessment expect the following signs and symptoms when
findings will include the following, one is under stress, EXCEPT___________
EXCEPT______________.
a. Blood loss and weakness
a. Dependent edema
b. Increases respiration rate
b. Pulmonary crackles
c. Decreased mobility
c. Difficulty of breathing
d. Pain due to tissue damage
40. Based on your knowledge, Patient Maya, b. Persistent pain
who has a history of cardiac illness, should not
be given an enema before surgery. Which of the c. Acute pain
following reasons inhibits the order of enema
for Patient Maya? Enema____________
d. Intermittent pain

a. Paralyzes the peristalsis movement and


43. In order for the nurse to recall the location
increases abdominal pain
of pain, he has to_______________.
b. Produces vagal stimulation that is
dangerous to cardiac patient a. Asks for onset and duration

c. Causes constipation and fecal impaction after b. Mark the painful area in a body diagram
the surgery

c. Asks for facial expression


d. Enema results to increased water absorption
in the bowels
d. Asks verbal description using pain intensity
scale
Situation: Mr. S came to the ER because of
sharp troubling pain. After his surgery, he 44. As example of a drug therapy to relieve
claimed pain is felt even he is asleep. moderate pain is_____________.

a. Codeine
41. At what stage of pain mechanism do you
b. Demerol
classify this pain?

c. Methadone
a. Perception
d. Morphine sulphate
b. Modulation
45. When a client is on prolonged pain therapy,
c. Transmission the nurse should watch for____________.

d. Transduction
a. Tolerance to drug
42. When a client complains of pain less than 6
b. Allergic reaction to drug
months, it is called_____________.

c. Drug resistance
a. Chronic pain
d. Addiction to drug c. QRS complex

Situation: Donny a 46 year old patient d. P wave


admitted to the coronary care unit (CCU) with
an MI and frequent premature ventricular 48. A considerable difference between the

contractions (PVCs) has doctor orders for apical and radial pulse rate of Donny would

continuous amiodarone infusion, IV indicate

nitroglycerin infusion, and morphine sulfate 2


mg IV every 10 minutes until there is relief of a. Stronger left than right ventricular muscles

pain. She is taken care by Leona a newly


b. Numerous weak ineffectual cardiac
graduate nurse.
contractions

c. Thickened myocardium and large heart


chambers
46. Because of Donny’s premature ventricular
contraction, the nurse should monitor its effects d. Increased pressure in systemic arteries
on which of the following parameters?
49. As Donny is assessed he complains of

a. Electrolyte levels being nauseated and very weak. The nurse


should
b. Apical radial heart rate
a. Perform nutritional assessment
c. Oxygen saturation
b. Alert staff for potential help
d. Medications
c. Explore and discuss possible effect of stress
47. In analyzing a patient’s
electrocardiographic (ECG) rhythm strip, Leona d. Provide reassurance while focusing on
uses the knowledge that the time of the pleasant topics
conduction of an impulse through the Purkinje
50. The Physician scheduled for an exercise
fibers is represented by
electrocardiogram (stress test). What
information should the nurse include when
a. PR interval
explaining the value of this test? Exercise

b. QT interval stress testing is a:


a. definitive method to diagnose the cause of passed earlier this morning”. In anticipation of
chest pain defecation, which of the following instructions
are most important for you, the nurse, to give to
b. diagnostic modality of minimal value in this client?
planning treatment of angina

A. Please call the nurse if you need to go to the


c. noninvasive means of assessing bathroom.
cardiovascular conduction and function

B. If you feel the urge to have a bowel


d. minimally invasive manner of assessing a movement, please call for assistance before
body’s reaction to increase in exercise getting up to the toilet. When having a bowel
movement, be sure to breathe out to prevent
Situation: You are a nurse tasked to care for straining. Do not hold your breath.
patients with several different conditions. You
utilize your knowledge of nursing concepts to C. To prevent the Valsalva maneuver, contract
the stomach muscles while holding your breath
help these patients.
and push. This will assist in the passage of the
stool and will decrease the amount of time
required to have a bowel movement.

51. The patient admitted in the unit with a D. Your bowels will be moving soon. Please
urinary condition asked you, the nurse, where in report any abdominal pain.
the kidney does urine get formed. You answer
them correctly by stating that urine is produced 53. You are the nurse on duty in the unit. A

in the: client verbalized complaints of a recent


constipation. You took the patient’s health
history. Which of the following statements by
A. Glomerulus.
the client suggests the likely cause of their
B. Proximal convoluted tubule. constipation?

C. Loop of Henle.
A. I walk with a group of friends every day at
the mall for an hour.
D. Nephron.
B. My spouse died 20 years ago, but my family
52. A client was assigned to your unit after their is very loving and supportive. They live just
abdominal surgery. You asked the patient around the corner and come over a few times a
week to visit.
during your morning rounds about the passage
of flatus. The patient answered, “Yes, flatus has
C. The fast food place near my home has history of renal insufficiency. They had been on
really good food. I eat there most of the fluid restriction and renal diet. Their laboratory
time.
shows a steady increase in BUN, creatinine,

D. What is a laxative? and potassium. The client’s spouse


accompanied the client to the appointment. She
54. You are the nurse on duty in the emergency pulled you aside and stated that his husband
room. A client came via ambulance with has been having episodes of confusion each
shortness of breath for the past 3 days. After a day. She told you that she is very concerned
few hours in the ER, the client is admitted to the about her husband and she wants to know if he
intensive care unit with pulmonary edema that is having small strokes. Based on the
requires intubation and ventilation. A Foley information provided, what is your best
catheter was placed in the client and he had a response to the spouse’s question?
total of 25 mL urine output. The laboratory
reveals: blood glucose of 300, blood urea A. Confusion is a common sign of transient
nitrogen of 100, and creatinine of 5.0. The client ischemic attacks. Thank you for informing me of
this. The client will need a CAT scan of the
has a history of CHF, CAD, diabetes, COPD,
head.
and asthma. What's the client’s most likely
cause of low urine output? B. The client’s kidneys are not working very
well. However, confusion is not a common
symptom. I will inform the physician of the
A. Acute and chronic renal failure due to
confusion and have her assess the situation
diabetes and a decreased blood flow to the
further with the client.
kidneys due to heart failure.

C. The elevated potassium is causing the


B. Renal failure due to decreased coronary
confusion. The client will need some medication
output secondary to heart failure.
to decrease the potassium level.

C. Decreased blood flow to the kidneys due to


D. The client is experiencing worsening
congestive heart failure (CHF) secondary to
uremic syndrome. This is associated with
noncompliance with home fluid restriction.
kidney failure and is a sign that the client’s
kidney function is becoming worse. I will
D. Severe dehydration. notify the physician about the confusion.
There are a couple of treatment options to
55. A client came to the hospital complaining of consider. The physician will discuss the
nausea and occasional vomiting. You are the treatment options with the client and you.

nurse reviewing the client’s medical records


56. One of the elderly patients assigned to you
when you note that this client has a 4 year
in the ward has been complaining of increasing
trips to the bathroom to urinate. Her estimated C. Avoid touching the tip of the spigot to any
coffee intake is 3 cups every day. What is the surfaces when emptying the collection bag.

best explanation you can provide to this


D. Encourage the client to drink at least 2000
patient? mL each day and carefully wash the perineal
area, with soap and water, at least twice
daily and with each bowel movement.
A. The increased urine production is most likely
due to a urinary tract infection.
58. A client of yours read the term activities of
B. Coffee is causing the increased urination due daily living. As a knowledgeable nurse, you
to your increased fluid intake. This is completely know that activities of daily living (ADLs) are the
normal and nothing to be concerned about.
essential and routine tasks that most young,
healthy individuals can perform without
C. Coffee is causing the increased urination.
Coffee contains caffeine that causes assistance. These include the basic activities
diuresis, or increased urine formation. that are performed in the course of a normal
Simply decreasing the number of cups of
day. You also know about instrumental activities
coffee you drink each day, and limiting the
consumption of caffeinated beverages to the of daily living (IADL). Which of the following
morning hours, should help decrease your statements best describes IADL?
trips to the bathroom.

A. Activities that are usually performed in the


D. Drinking coffee increases the circulating
course of a normal day. These activities include
plasma in the body and this increases the urine
ambulating, eating, dressing, bathing, brushing
formation. Simply decreasing the number of
the teeth, and grooming.
cups of coffee you are drinking should help.

B. Activities that assist the client in recognizing


57. You are beginning your shift for the day. You
and managing stress. These activities include
start by assessing a client that has a Foley facilitating interpersonal relationships, allowing
catheter connected to a collection bag. Which adequate time for rest, and providing regular,
of the following is the best routine catheter care nutritious meals.

actions to take while caring for this client?


C. Activities that allow the client to be
independent in society. These activities
A. Encourage increased oral fluid intake and include shopping, preparing meals, paying
observe for any opacity in the urine suggesting bills, and taking medications appropriately.
bacterial infection.
D. Activities that support the effectiveness of
B. Carefully wash the perineal area with soap direct care interventions. These activities
and water after each bowel movement. include checking equipment, directing the
maintenance of the client’s room, and managing
the supply of materials needed for client care.
59. One of your clients arrived in the A. I.
preoperative area for their knee surgery. You
asked them to put an elastic stocking on the B. I, II, III, IV

non operative leg. The client asked you, “What


C. I, II, IV
is the purpose of these stockings?”. Your best
response to the client’s question is: D. II, III, IV

A. The stockings promote return of venous Situation: You are a nurse providing palliative
blood to the heart and assist in preventing care to Coby, an elderly with terminal illness.
the blood from clotting in the legs. You use your knowledge on terminal illness and
palliative care to provide the best care to this
B. The operating room is very cold. The
stockings assist in maintaining a healthy core patient.
body temperature during the operation.

C. The stockings promote joint mobility.


61. Coby approached you and asked about
D. The stockings promote the return of arterial palliative care since he has been hearing it a lot
blood to the heart and prevent blood from lately after he was diagnosed with his terminal
clotting in the legs.
illness. As a knowledgeable nurse, you know
which of the following is the best definition of
60. The relative of an immobile client in your
palliative care?61. Coby approached you and
unit asked you, the nurse, about the
asked about palliative care since he has been
complications of immobility. You answer her
hearing it a lot lately after he was diagnosed
correctly by stating that which of the following
with his terminal illness. As a knowledgeable
are the complications of immobility? Select all
nurse, you know which of the following is the
that apply.
best definition of palliative care?

I. Primary osteoporosis.
A. Care for terminally ill clients.

II. Foot drop. B. Symptom management for a client when a


disease no longer responds to cure-focused
treatment.
III. Urinary stasis.

C. Aggressive cure-focused disease treatment


IV. Pressure ulcer. and management.
D. Comfort care. C. Provide education to the patient and family
regarding oral care and antiemetic medication.
62. Coby asked you what the goal of palliative
care is. You answer him correctly by stating that D. Take a detailed medical history to determine
the cause of the nausea.
which of the following are the goals of palliative
care? Select all that apply. 64. The granddaughter of Coby approached
you and asked regarding the symptoms their
I. Preventing disease symptoms. grandfather might experience. You answer her
correctly by stating that which of the following
II. Relieving disease symptoms. are the common symptoms of terminally ill
clients?
III. Curing a disease.
A. Hunger, thirst, fatigue, and diarrhea.
IV. Treating a disease
B. Dehydration, nausea, effective breathing,
and adequate nutrition.
A. I
C. Discomfort, nausea, ineffective breathing,
B. II and fatigue.

C. I, II D. Urinary continence, thirst, dehydration, and


diarrhea.
D. III, IV
65. Coby is being cared for at home by his
63. Coby is now under home health with family members. You conducted your physical
palliative care services. He stated that he has assessment on Coby and based on your
been experiencing nausea. Which of the findings, you are aware that Coby’s death is
following actions will most likely promote imminent. What is your most important role in
comfort in this client? the care of family at this point in time?

A. Educate the patient and family in the use of A. Providing temporary relief of caregiving
prescribed antiemetics; providing oral care duties to allow the family to rest.
every 2 to 4 hours; consuming a diet of clear
liquids and ice chips; and avoiding liquids such
B. Providing education regarding the symptoms
as coffee, milk, and citrus juices.
the client will likely experience.

B. Administer additional pain medication.


C. Coordinating a visiting schedule for the B. High serum lipid levels.
family.
C. Low serum potassium level.
D. Communicating news of the client’s
impending death to the family while they are D. Low serum lipid levels.
together.
68. You are conducting your assessment on
Situation: You are a nurse tasked to care for
Alvida. You are checking her lower extremities
Alvida, a patient with peripheral vascular
expecting to find which of the following clinical
disease. You use your knowledge on PVD to
manifestations of peripheral vascular disease?
help care for this patient effectively.

A. Hairy legs.

66. You are planning care for patient Alvida with B. Mottled skin.
a history of PVD with symptoms of claudication.
C. Pink, cool skin.
The focus of your nursing care should be
directed in avoiding which of the following
D. Warm, moist skin.
scenarios?
69. Your patient Alvida with PVD has
A. Oxygen demand by the muscle exceeds undergone a right femoral-popliteal bypass
the supply. graft. You assess her blood pressure noting a
decrease from 124/80 to 94/62. Which of the
B. Oxygen demand and supply of the working
following should you assess in Alvida first?
muscle are in balance.

C. Oxygen supply exceeds the demand of the A. IV fluid solution.


working muscle.
B. Pedal pulses.
D. Oxygen is absent.
C. Nasal cannula flow rate.
67. You are reviewing the labs of Alvida. You
note which of the following common abnormal D. Capillary refill.
laboratory results that are associated with the
development of peripheral vascular disease 70. After further assessment on Alvida who was

(PVD)? diagnosed with PVD, you found out that she


also has a history of heart failure. You will
develop a plan of care for Alvida based on the
A. High serum calcium level.
fact that she may have low tolerance for Based on the data and expected outcomes,
exercise related to: which of the following should you emphasize in
the teaching plan for Morgan?
A. Decreased blood flow.
A. Food intake.
B. Increased blood flow.
B. Fluid volume.
C. Decreased pain.
C. Skin integrity.
D. Increased blood viscosity
D. Tissue perfusion.
Situation: You are a nurse in the cardiothoracic
ward of Hospital Merry. You are assigned to 72. Helmeppo, a client admitted to the
patients with aneurysms. You will utilize your emergency department is complaining of
nursing knowledge on aneurysms to care for severe abdominal pain. After several tests, a
these patients. radiograph of his revealed a large abdominal
aortic aneurysm. The primary goal for
Helmeppo at this time is to:

71. You are developing a discharge teaching


plan for Morgan, a patient who underwent a A. Maintain circulation.

repair of abdominal aortic aneurysm a few days


B. Manage pain.
ago. You reviewed Morgan’s chart for
information about his health history. Key C. Prepare the client for emergency surgery.
findings you noted in his chart are as follows:
D. Teach postoperative breathing exercises.

1) Smokes 4 cigars a month.


73. Yasopp, a 54 year old client was admitted in
the emergency department. On assessment, it
2) Vital signs: blood pressure, ranges from
was revealed he has severe back pain, Grey
150/76 mm Hg to 170/98 mm Hg; heart rate, 90
Turner’s sign, nausea, BP of 90/40, HR of 128
to 100 beats per minute; respirations, 12–18
bpm, and RR of 28 cpm. As Yasopp’s nurse,
per minute; temperature, 99.9° F (37.8° C).
you should first do which of the following
actions:
3) +1 bilateral ankle edema.

A. Assess the urine output.


B. Place a large bore I.V. D. An X-ray examination.

C. Position onto the left side. Situation: You are tasked to care and educate
patients regarding oral care to help their
D. Insert a nasogastric tube. conditions. You utilize your knowledge to assist
these patients.
74. Arlong, one of the patients assigned to you,
complains of sudden, severe pain in his back
and chest, accompanied by SOB. He describes
the pain sensation as “as if something was 76. Buggy, a client admitted to your unit, has
tearing inside”. The physician suspects that he stomatitis. As a knowledgeable nurse, you
is experiencing a dissecting aortic aneurysm. know that which of the following interventions is
The code cart is brought into Arlong’s room most appropriate for Buggy at this time?
because you know that one of the
complications of dissecting aneurysm is: A. Drinking hot tea at frequent intervals.

B. Gargling with antiseptic mouthwash.


A. Cardiac tamponade.

C. Using an electric toothbrush.


B. Stroke.

D. Eating a soft, bland diet.


C. Pulmonary edema.

77. You are doing your assessment on patient


D. Myocardial infarction.
Kuro, and when you check his mouth, you note
75. Hatchan is to be discharged after his the absence of saliva. Further assessment
surgery of aortic aneurysm repair with synthetic reveals he has pain in the area of his ear. Kuro
graft to replace part of his aorta. As part of your has been NPO for several days now because of
discharge teaching, you instruct Hatchan to an NGT insertion. Based on your assessment,
notify his physician before doing which of the you suspect that Kuro may be developing which
following procedures? of the following conditions of the mouth?

A. Blood drawn. A. Stomatitis.

B. An I.V. line inserted. B. Oral candidiasis.

C. Major dental work. C. Parotitis.


D. Gingivitis. D. “It is good that you can talk about your
concerns. Try calling a friend when you want
78. You are tasked by your manager to conduct to smoke.”

a presentation to the community regarding oral


80. Kuro was rushed to the emergency
cancer. You conducted your research, and you
department after a motor vehicle accident
found out several risk factors for the disease.
where he fractured his mandible. Surgery has
Which of the following will you include in your
been performed to immobilize his injury. The
presentation as the primary risk factor for oral
surgeon has wired Kuro’s jaw. In the immediate
cancer?
postoperative phase, you as the nurse should:

A. Use of alcohol.
A. Prevent nausea and vomiting.

B. Frequent use of mouthwash.


B. Maintain a patent airway.

C. Lack of vitamin B12.


C. Provide frequent oral hygiene.

D. Lack of regular teeth cleaning by a dentist


D. Establish a way for the client to
communicate.
79. Jango is one of your patients in the unit. He
has entered a smoking cessation program to
Situation: You are a nurse caring for Tashigi, a
quit a 2 pack per day cigarette habit of his. He
patient with a chronic condition of the
tells you, “I have not smoked a cigarette for 3
respiratory system. You help her by utilizing
weeks, but I am afraid I am going to slip up and
your nursing knowledge and skills on the topic.
smoke because of the pressures of my current
job”. As Jango’s nurse, what would be the most
appropriate reply to make in response to his
81. You are visiting Tashigi who reports that her
comments?
chronic bronchitis has recently worsened.
Which of the following instructions would you
A. “Don’t worry about it. Everybody has difficulty
quitting smoking, and you should expect to as reinforce to Tashigi to help with her condition?
well.”
A. Increase amount of bedrest
B. “If you increase your self-control, I am sure
you will be able to avoid smoking.”
B. Increase fluid intake

C. “Try taking a couple of days of vacation to


C. Decrease caloric intake
relieve the stress of your job.”
D. Reduce home oxygen use decreasing. Which of the following would be
your most appropriate intervention?
82. You are completing an assessment on
Tashigi. Based on your knowledge and previous
A. Place the client in the prone position
experience of working with patients who have
chronic bronchitis, which of the following B. Notify the respiratory therapist to increase
findings would you expect Tashigi to present? the positive pressure settings

C. Call the physician to suggest sedatives


A. Minimal sputum with cough and paralytics

B. Pink, frothy sputum D. Prepare to administer intravenous


aminophylline
C. Barrel chest
85. You are doing your charts in the nurses’
D. Stridor on expiration
station when you suddenly hear high pressure
alarms in Tashigi’s room. Which of the following
83. Tashigi has been learning more about self
is your next best action?
care for her chronic condition at a community
health class. She asked you why the
A. Wait and allow the client time to regulate
participants are being taught about lip
breathing in coordination with the ventilator
breathing. You respond correctly to Tashigi by
explaining that pursed lip breathing will help to: B. Check ventilator tubing and connections

A. reduce upper airway inflammation. C. Silence the alarm and restart the ventilator

B. reduce anxiety through humor. D. Lower the tidal volumes being delivered to
the client

C. strengthen respiratory muscles.


Situation: You are a nurse knowledgeable

D. increase effectiveness of inhaled about various ethical, legal, and safety issues in
medications nursing as this will guide you in establishing a
safe nursing practice.
84. Tashigi’s condition worsened lately and she
now needs a positive pressure mechanical
ventilation. She has been fighting the ventilator
86. Nojiko is a hospitalized client under your
assisted breaths, and her BP has been steadily
care. She was diagnosed with end stage cancer
and has suddenly decided to discontinue her oxygen. What is the key ethical principle
treatment. She requests no more additional involved in this scenario?
treatment like antibiotics, tube feedings, and
mechanical ventilation. Acting as Nojiko’s A. Nonmaleficence
advocate, which of the following actions should
you take? B. Fidelity

C. Beneficence
A. Respect the client’s wishes and indicate
those wishes on the plan of care
D. Justice

B. Encourage the client to share the


88. Van, one of the patients in your unit, has a
decision with the family and the client’s
physician sudden change in his condition. You called the
physician to report this occurrence. He gave
C. Clarify other treatments that the client wishes orders over the telephone for ABGs to be drawn
to withhold
stat. Which of the following is the most
important safety consideration when obtaining
D. Wait until additional treatment is required
and then decide what to do based on the the physician’s order over the phone?
client’s condition

A. Writing the order down and reading it


87. A car-pedestrian accident occurred nearby. back to the physician
The pedestrian client named Mohji was brought
to the ER of the hospital you are working on. B. Calling the respiratory therapist stat to draw
Mohji is alert and oriented but complains of the ABGs

difficulty breathing. His SpO2 levels vary from


C. Giving the order stat to the health unit
88-90%. O2 was applied at 2L per nasal coordinator to place in the computer
cannula with no improvement in SpO2. Oxygen
per mask is then initiated at 40% with little D. Writing down the order for ABGs immediately
improvement. After some tests, Mohji’s
89. Patient Cabaji is to be admitted to the
radiograph films reveal no obvious injuries or
surgical unit. He has multiple rings, a bracelet,
fractures. Suddenly, Mohji loses consciousness,
a watch, and PHP 3500 in cash. Which of the
has a respiratory arrest, and subsequently dies.
following is the safest action for you to take
During his resuscitation, it is determined that
regarding the patient’s valuables?
one of the nurses failed to open the valve to the
O2 tank and Mohji has not been receiving
A. Allowing the client to keep the items so they diseases to effectively plan and manage care
will be safeguarded by the client for these patients.

B. Collecting the items and placing them in the


client’s room closet
91. Patient Bellmere was admitted to the

C. Giving the money to the client’s spouse and hospital with a diagnosis of nonalcoholic fatty
allowing the client to keep the jewelry liver disease (NAFLD). You are reviewing her
chart for her health history. Which of the
D. Collecting the items according to hospital following findings is consistent with the NAFLD
policy for safekeeping
disease process?

90. You admit patient Kuroobi who is


complaining of nausea and vomiting to the A. 70 years old
emergency department. Kuroobi is alone in the
B. Obese
ED without any relatives or guardians. After you
complete your assessment on Kuroobi, you
C. History of recent antibiotic use
prepare to leave the room. Which of the
following statements is your safest instruction D. Living in colder climates
for Kuroobi?
92. Yosaku is a male patient diagnosed with
cirrhosis. While caring for Yosaku, you add the
A. “If you need to vomit, here is a basin for you.
I don’t want you to get up on your own.” nursing diagnosis Disturbed body image related
to physical manifestations of the illness when
B. “I will be in the room next door. I’ll check you overheard Yosaku telling his brother:
back in about 10 minutes.”

C. “I will go update the doctor about you. Do A. “I don’t think I can handle this disease.”
you need anything before I go?”
B. “I know the doctors say I have liver failure,
D. “Here is the nurse call light. Press this but I don’t really believe them.”
button if you need me.”
C. “I know I should rest more, but I’m just not
Situation: You are a nurse assigned to take that type of person.”

care of patients with various liver conditions.


D. “I don’t like the fact that I seem to have
You utilize your knowledge and skills on the breasts now.”
93. Patient Johnny was admitted in the ED after C. administering antibiotics.
complaints of upper right sided abdominal pain.
You suspect that the client may have liver D. encouraging frequent ambulation.
cancer when which serum laboratory test result
is noted to be elevated?
Situation: You are a nurse providing care for
patient Vivi who came to the hospital for
A. Creatinine
assessment. She is suspected to have
B. Serum α-fetoprotein (AFP) levels osteoporosis. You help her by utilizing your
knowledge on the topic.
C. Serum phosphorus levels

D. CA-125
96. You reviewed Vivi’s chart after conducting
94. You are caring for patient Carol after her her physical assessment. You correctly identify
liver biopsy with the assistance of student nurse that which of the following signs/symptoms
Apis. You evaluate that Apis understands liver indicate that Vivi has already developed
biopsy post procedure care when she does osteoporosis?
which of the following?

A. The client has lost one (1) inch in height.


A. plans to monitor vital signs every hour.
B. The client has lost 12 pounds in the last year.
B. promotes ambulation 1 hour after the
procedure. C. The client’s hands are painful to the touch.

C. positions the client on the right side. D. The client’s serum uric acid level is elevated.

D. encourages the client to cough and deep 97. After Vivi’s diagnosis of osteoporosis, she
breathe immediately following the procedure.
asked you why smoking cigarettes causes her
bones to become brittle. Your most appropriate
95. Pell is a patient hospitalized for
response is:
conservative treatment of liver cirrhosis. As part
of the collaborative plan of care, you would
A. “Smoking causes nutritional deficiencies,
anticipate which of the following?
which contribute to osteoporosis.”

A. monitoring the client’s blood sugar. B. “Tobacco causes an increase in blood supply
to the bones, causing osteoporosis.”
B. maintaining NPO (nothing by mouth) status.
C. “Smoking low-tar cigarettes will not cause of the following health teaching should you
your bones to become brittle.” implement for Vivi?

D. “Nicotine impairs the absorption of


calcium, causing decreased bone strength.” A. Encourage the client to take Tums with at
least eight (8) ounces of water.
98. When discussing osteoporosis with Vivi, you
B. Teach the client to take Tums with the
are aware that which of the following is an
breakfast meal only.
example of a secondary nursing intervention?
C. Instruct the client to take Tums 30 to 60
A. Obtain a bone density evaluation test. minutes before a meal.

B. Perform non–weight-bearing exercises D. Discuss the need to get a monthly serum


regularly. calcium level.

C. Increase the intake of dietary calcium.


RECALLS 7 - NP4

D. Refer clients to a smoking cessation


Situation: You are caring for patient Igaram, a
program.
patient with Addison’s disease. You utilize your
99. Vivi is prescribed by her physician knowledge on this concept to help care for the
Calcitonin by nasal spray. Which of the patient.
following assessment findings indicate an
adverse effect of the medication?

1. Igaram was just recently diagnosed with


A. The client complains of nausea and vomiting. Addison’s disease. He still lacks knowledge
about his disease so he decided to ask you
B. The client is drinking two (2) glasses of milk
some questions. Igaram asked you, “How does
a day.
the disease happen?” You answer him correctly
C. The client has a runny nose and nasal by stating that this disease results from:
itching.

A. Insufficient secretion of growth hormone


D. The client has had numerous episodes of
(GH).
nosebleeds.

B. Dysfunction of the hypothalamic pituitary.


100. Vivi is taking Tums to help prevent the
further development of her osteoporosis. Which C. Idiopathic atrophy of the adrenal gland.
D. Oversecretion of the adrenal medulla. adequately managed. Which of the following
manifestations would be expected in Igaram if
2. Igaram is admitted to your unit. After your he develops this condition?
assessment on him, you formulated the nursing
diagnosis Deficient fluid volume related to
A. Fluid retention.
inadequate fluid intake and to fluid loss
secondary to inadequate adrenal hormone B. Pain.
secretion. As Igaram’s oral intake increases,
which of the following fluids would be the most C. Peripheral edema.

appropriate for him?


D. Hunger.

A. Milk and diet soda. 5. If Igaram develops Addisonian crisis, which


of the following would be your priority as
B. Water and eggnog.
Igaram’s primary nurse?

C. Bouillon and juice.


A. Controlling hypertension.
D. Coffee and milkshakes.
B. Preventing irreversible shock.
3. You are instructing Igaram how to adjust the
dose of the glucocorticoids he is taking. As his C. Preventing infection.

nurse, you should explain to him that he may


D. Relieving anxiety.
need an increased dosage of glucocorticoids in
which of the following scenarios? Situation: You are a nurse tasked to care for
patients with different pituitary disorders. You
A. Completing the spring semester of school. are to take care of Karoo and Koza, both
diagnosed with SIADH, and Paula, who is
B. Gaining 4 pounds.
diagnosed with diabetes insipidus. You utilize

C. Becoming engaged. your knowledge on this concept to help care for


your patients safely.
D. Undergoing a root canal.

4. Igaram is diagnosed with Addison’s disease.


6. One of the clients in your unit, Karoo, is
As a knowledgeable nurse, you know that this
diagnosed to have a pituitary tumor. Karoo
condition may lead to Addisonian crisis if not
developed Syndrome of Inappropriate
Antidiuretic Hormone (SIADH). Which of the not care if he dies. Which of the following
following interventions should you implement as actions by the nurse is an example of the
Karoo’s primary nurse? ethical principle of autonomy?

A. Assess for dehydration and monitor blood A. Discuss the information the client told the
glucose levels. nurse with the health-care provider and
significant other.
B. Assess for nausea and vomiting and
weigh daily. B. Explain it is possible the client could have a
seizure if he drank fluid beyond the restrictions.
C. Monitor potassium levels and encourage
fluid intake. C. Notify the health-care provider of the
client’s wishes and give the client fluids as
D. Administer vasopressin IV and conduct a desired.
fluid deprivation test.
D. Allow the client an extra drink of water and
7. You are reviewing the chart of Karoo who has explain the nurse could get into trouble if the
client tells the health-care provider.
SIADH. Which of the following clinical
manifestations you noted in Karoo’s chart
9. Paula, another patient assigned to you in the
should be reported to his primary care
unit, is recently diagnosed with diabetes
physician?
insipidus. Which of the following interventions
should you implement as Paula’s primary
A. Serum sodium of 112 mEq/L and a nurse?
headache.

B. Serum potassium of 5.0 mEq/L and a A. Administer sliding-scale insulin as ordered.


heightened awareness.
B. Restrict caffeinated beverages.
C. Serum calcium of 10 mg/dL and tented
tissue turgor. C. Check urine ketones if blood glucose is
>250.
D. Serum magnesium of 1.2 mg/dL and large
urinary output. D. Assess tissue turgor every four (4) hours

8. Another patient in your unit, Koza, was 10. Following Paula’s diagnosis of diabetes
diagnosed with SIADH secondary to cancer of insipidus, she stayed in the hospital for a few
the lung. He tells you that he wants to days. She is now about to be discharged and
discontinue his fluid restriction and that he does you are conducting your health teaching
regarding her condition. Which of the following 12. Pierre had a recent exacerbation of
statements made by Paula warrants further ulcerative colitis. He is put on mesalamine
(Asacol), which is to be administered rectally via
intervention?
an enema. Pierre finds this procedure
distasteful and he asks you, “Why can't the
A. “I will keep a list of my medications in my medication just be given orally?”. You answer
wallet and wear a Medic Alert bracelet.” Pierre correctly by saying which of the
following?

B. “I should take my medication in the


morning and leave it refrigerated at home.” A. “It can be given orally; I’ll contact the doctor
and see if the change can be made.”

C. “I should weigh myself every morning and


record any weight gain.” B. “Rectal administration delivers the
medication directly to the affected area.”

D. “If I develop a tightness in my chest, I will call


my health-care provider.” C. “Oral administration will not be as effective
for the disease condition.”

Situation: You are a nurse caring for patients


D. “It can be given orally, I’ll make the change
with inflammatory bowel disease. and we’ll tell the doctor in the morning.”

13. You overhear a licensed practical nurse


(LPN) talking to one of your patients, Mousse,
11. Pierre is a patient admitted to the hospital
who is being prepared for a total colectomy with
with a diagnosis of ulcerative colitis. You are
creation of an ileoanal reservoir for her
currently reviewing his history and physical
ulcerative colitis. To decrease Mousse’s anxiety,
assessment chart. Based on Pierre’s diagnosis,
you should intervene to clarify the information
which of the following information should you
given by the LPN when you hear the LPN
expect to see in Pierre’s medical records?
saying:

A. Abdominal pain and bloody diarrhea


A. “This surgery will prevent you from
developing colon cancer.”
B. Weight gain and elevated blood glucose

B. “After this surgery you will no longer have


C. Abdominal distension and hypoactive bowel
ulcerative colitis.”
sounds

C. “When you return from surgery you will not


D. Heartburn and regurgitation
be able to eat solid food for several days.”
D. “You will have an ileostomy when you D. fluid imbalance.
return from this surgery.”
Situation: You are a new nurse assigned to
14. Wyper, a 20 year old male client, is admitted take care of patients with various eye disorders.
to your unit because of the exacerbation of their You use your knowledge to help these patients
ulcerative colitis. You go into Wyper’s room to with their condition.
complete an initial assessment, and he yells,
“Get outta here! I am tired of you nurses and
doctors looking at my body all the time!” Which
16. One of the patients in your unit, Pagaya, is
of the following is your best action?
diagnosed with glaucoma. Which of the
following symptoms should you expect the
A. Leave the room and ask a male colleague to
client to report during your initial assessment
complete the assessment.
with him?
B. Verbally acknowledge the client’s
frustration and anger.
A. Loss of peripheral vision.

C. Call the health-care practitioner and ask for a


B. Floating spots in the vision.
sedative order.

C. A yellow haze around everything.


D. Tell the client that gathering data about his
current condition will promote effective timely
treatment of his health concerns D. A curtain coming across vision.

15. You are caring for Conis, a patient admitted 17. Pagaya has now been prescribed a miotic

in your unit who is diagnosed with Crohn’s cholinergic medication for his glaucoma. Which

disease. She has undergone a barium enema of the following data indicates that the

that demonstrated the presence of strictures in medication has been effective on Pagaya?

her ileum. Based on this finding, you should


monitor the client closely for signs of: A. No redness or irritation of the eyes

B. A decrease in intraocular pressure


A. peritonitis.

C. The pupil reacts briskly to light


B. obstruction.

D. The client denies any type of floaters


C. malabsorption.
18. You are caring for Gan Fall, a postoperative need to clean my glasses all the time”. Which of
patient, after his retinal detachment surgery. the following eye disorders should you suspect
Gas tamponade was used to flatten the
that Eneru has?
patient’s retina during the procedure. Which of
the following interventions should you
implement first? A. Corneal dystrophy

A. Teach the signs of increased intraocular B. Conjunctivitis


pressure.

C. Diabetic retinopathy
B. Position the client as prescribed by the
surgeon.
D. Cataracts

C. Assess the eye for signs/symptoms of


Situation: You are a nurse studying the
complications.
different types of shock and its appropriate
D. Explain the importance of follow-up visits. nursing interventions. You come across the
following patients in your unit. You applied the
19. You are caring for Conis, a patient with concepts you’ve learned to your nursing
severe myopia. She is scheduled for a laser practice.
assisted in situ keratomileusis (LASIK) surgery.
Which of the following instructions should you
discuss with Conis prior to her discharge from
21. Foxy is a client admitted to the emergency
the surgery?
department. Assessment findings include
diaphoresis, pale clammy skin, and a blood
A. Wear bilateral eye patches for three (3) days.
pressure reading of 90/70. Which of the

B. Wear corrective lenses until the follow-up following interventions should you implement
visit. first?

C. Do not read any material for at least one (1)


A. Start an IV with an 18-gauge catheter.
week.

B. Administer dopamine intravenous infusion.


D. Teach the client how to instill
corticosteroid ophthalmic drops.
C. Obtain arterial blood gases (ABGs).
20. Eneru, a 65 year old male client is
D. Insert an indwelling urinary catheter.
complaining of blurred vision, but denies having
any type of pain. He reports to you, “I feel like I
22. Porche is a patient diagnosed with C. Apply sequential compression devices to the
neurogenic shock. As a knowledgeable nurse, lower extremities.
you expect to note which of the following signs
and symptoms in this client? D. Administer an antipyretic medication
every four (4) hours PRN.
A. Cool, moist skin.
25. A patient named Chiqicheetah presents
B. Bradycardia. themselves in the emergency department
complaining of abdominal pain, is pale and
C. Wheezing.
clammy, and has a pulse of 110 and a blood

D. Decreased bowel sounds pressure reading of 92/60. Chiqicheetah has


vertebral fractures, and she reported she has
23. One of the patients in your unit, Hamburg, been self-medicating with Ibuprofen, a type of
was diagnosed with septicemia. The following nonsteroidal anti-inflammatory drug (NSAID).
are the orders given by Hamburg’s primary Which of the following type of shocks should
physician. Which of these orders will have the you expect in patient Chiqicheetah?\
highest priority?

A. Cardiogenic shock.
A. Provide a clear liquid diet.
B. Hypovolemic shock.
B. Initiate IV antibiotic therapy.
C. Neurogenic shock.
C. Obtain a STAT chest x-ray.
D. Septic shock.
D. Perform hourly glucometer checks.
Situation: You are caring for patients in your
24. You wrote the nursing diagnosis of unit with alterations in their fluid and
“alteration in comfort related to chills in fever” in electrolytes. As a knowledgeable nurse, you
one of your patients who has sepsis. Which of apply the concepts of fluid and electrolytes in
the following interventions would you include in your nursing practice.
this patient’s plan of care?

A. Ambulate the client in the hallway every shift.


26. As an experienced nurse, you know that
client incidence of hypermagnesemia is rare in
B. Monitor urinalysis, creatinine level, and BUN
level. comparison with hypomagnesemia. A student
nurse approached you and asked how
hypermagnesemia develops. You answer her D. A client receiving frequent wound irrigations
correctly by saying that hypermagnesemia
generally occurs secondary to: 29. You are refreshing your knowledge on
sodium imbalances. As a knowledgeable nurse,
you know that which of the following patients in
A. Cardiac contractility.
the ward is most likely to develop a sodium
B. Hypokalemia. level at 130 mEq/L (130 mmol/L)?

C. Liver failure. A. The client who is taking diuretics

D. Renal insufficiency. B. The client with hyperaldosteronism

27. You are assigned to care for Ace, a patient C. The client with Cushing’s syndrome
diagnosed to have hypokalemia. As a
knowledgeable nurse, you know that the D. The client who is taking corticosteroids
electrolyte that must be corrected in this
scenario is: 30. You are reviewing one of your patient’s
progress notes. You read that the physician has
documented “insensible fluid loss of
A. Calcium.
approximately 800mL daily”. As a
B. Magnesium. knowledgeable nurse, you make a notation that
insensible fluid loss occurs through which of the
C. Manganese. following types of excretion?

D. Zinc.
A. Urinary output
28. You are caring for a group of patients in the
B. Wound drainage
ward. While reviewing each of the patient’s
charts, you determine which of the following C. Integumentary output
patients is most likely at risk for fluid volume
deficit? D. The gastrointestinal tract

Situation: You are a nurse assigned to care for


A. A client with an ileostomy
and educate patients in the cancer unit of the
B. A client with heart failure hospital you are working on. You apply the

C. A client on long-term corticosteroid therapy


concepts you’ve learned regarding cancer to answer Bellamy correctly when you respond by
ensure a safe nursing practice. saying:

A. “CT is far superior to magnetic resonance


imaging for evaluating lymph node metastasis.”
31. You are reviewing your notes on cancer.
After much reading, you know that cancer B. “CT is noninvasive and readily available.”

prevalence is defined as?


C. “CT is useful for distinguishing small
differences in tissue density and detecting
A. The likelihood cancer will occur in a lifetime. nodal involvement.”

B. The number of persons with cancer at a D. “CT can distinguish malignant adenopathy
given point in time. from nonmalignant adenopathy.”

C. The number of new cancers in a year. 34. You are caring for Cricket, a patient with
pain related to bone cancer. You conducted an
D. All cancer cases more than 5 years old. assessment on Cricket in relation to this. You
know that which of the following is the most
32. Gol is your patient diagnosed to have
important component of a thorough pain
testicular cancer. He expressed his concerns
assessment specific for patient Cricket?
regarding fertility since him and his partner
desires to eventually have a family. As Gol’s
A. Intensity.
primary nurse, you discuss the option of sperm
banking. You inform Gol and his partner that
B. Cause.
sperm banking needs to be performed when?
C. Aggravating factors.
A. Before treatment is started.
D. Location.
B. Once the client is tolerating the treatment.
35. Noland is a cancer patient you are tasked to
C. Upon completion of treatment. care for. He is receiving the medication
vincristine (Oncovin). You plan your health
D. When tumor markers drop to normal levels. teaching for Noland regarding this medication.
Which of the following should you include in
33. You are working with Bellamy, a client with
known risks for lung cancer. He asks you why your instructions to Nolan?
he is scheduled for a computed tomography
(CT) scan as part of his initial workup. You
A. Use of loperamide (Imodium). D. Apply deodorant only under the left arm.

38. Tony Tony is a patient receiving radiation


B. Fluid restriction.
therapy for lung cancer. He complains that he is
C. Low fiber, bland diet. having difficulty sleeping. After hearing this
statement from your patient, you should:
D. Bowel regimen.

A. Suggest the client stop watching television


36. Sarquiss is a 57 year old client receiving
before bed.
chemotherapy that has the potential to cause
pulmonary toxicity. Which of the following B. Assess the client’s usual sleep patterns,
symptoms would you note in Sarquiss that amount of sleep, and bedtime rituals.

could indicate a toxic response to the


C. Tell the client sleeplessness is expected with
chemotherapy?
radiation therapy.

A. Decrease in appetite. D. Suggest that the client stop drinking coffee


until the therapy is completed.
B. Drowsiness.
39. You are caring for Nico, a patient with
C. Spasms of the diaphragm. cancer who requires a bolus tube feeding. You
prepare to administer the bolus tube feeding
D. Cough and shortness of breath. and as a skilled nurse, which of the following
nursing interventions is most appropriate to
37. Hina is one of the patients you are tasked to
decrease the risk of aspiration in this patient?
care for her in the unit. She is beginning
external beam radiation therapy to the right
A. Place the client on bed rest with the head of
axilla after her lumpectomy for breast cancer.
the bed elevated to 60 degrees for 2 hours.
You plan to conduct a health teaching. Which of
the following would you include in your B. Place the client on the left side with the head
education to Hina? of the bed at 45 degrees for 15 minutes.

C. Assist the client out of bed to sit upright


A. Use a heating pad under the right arm. in a chair for 1 hour.

B. Immobilize the right arm. D. Ask the client to rest in bed with the head of
the bed elevated to 30 degrees for 20 minutes.
C. Place ice on the area after each treatment.
40. Portgas is a cancer patient receiving C. Start an intravenous (IV) infusion of 0.9%
chemotherapy. He is experiencing a flare up of NaCl.

pruritus. You are planning to develop a care


D. Ask the client to sign a consent form.
plan for Portgas. In order to develop the nursing
care plan, you should ask him if he has been: 42. You are working with Dorry, a male patient
who experienced a myocardial infarction a few
A. Wearing clothes made from 100% cotton. days ago. You noted that patient Dorry seems
unusually fatigued. Upon your assessment, you
B. Sleeping in a cool, humidified room.
find that patient Dorry is dyspneic with activity,
has a heart rate of 110 bpm, and has
C. Increasing fluid intake to at least 3,000 mL/
day. generalized edema. Which of the following
actions would be most appropriate for this
D. Taking daily baths with a deodorant soap. patient?

A. Administer high-flow oxygen


Situation: You are a nurse caring for patients
B. Encourage the client to rest more
with cardiac complications. You use your
knowledge on cardiovascular concepts to help C. Continue to monitor the client’s heart rhythm
these patients.
D. Compare the client’s admission weight
with the client’s current weight

41. You are working on the unit with patient 43. You are caring for patient Whitebeard
Brogy. At 7:30 AM, you received a verbal order immediately following an insertion of a
from his primary HCP for a cardiac permanent pacemaker via his right subclavian
catheterization to be completed on him by 2:00 vein. As a skilled nurse, you know that the
PM. Which of the following actions should you action that can best prevent pacemaker lead
initiate first in Brogy? dislodgement is:

A. Initiate NPO (nothing per mouth) status A. inspecting the incision site dressing for
for the client. bleeding and the incision for approximation

B. Teach the client about the procedure. B. limiting the client’s right arm activity and
preventing the client reaching above
shoulder level
C. assisting the client with getting out of bed Situation: You are an emergency nurse tasked
and ambulating with a walker to work with patients with medical emergencies.
You utilize your knowledge to help these
D. ordering a stat chest x-ray following return
from the implant procedure patients.

44. You are increasing activity for patient Bon


with an admitting diagnosis of acute coronary
46. A patient’s wife is allowed to be present
syndrome. Which of the following symptoms
during resuscitation efforts for a patient in the
experienced by patient Bon best supports the
ICU. Which of the following statements made by
nursing diagnosis of activity intolerance?
you would be the most correct and
appropriate?]
A. Pulse rate increased by 15 beats per minute
during activity
A. “You can hold your loved one’s hand;
sometimes a recovering person remembers that
B. Blood pressure (BP) 130/86 mm Hg before
touch.”
activity; BP 108/66 mm Hg during activity

B. “Another staff member will be with you; I


C. Increased dyspnea and diaphoresis relieved
will show you where you can stand near
when sitting in a chair
your husband.”

D. A mean arterial pressure (MAP) of 80


C. “Because the resuscitation team needs to
following activity
work quickly, you need to stay out of their way
and not \ interfere.”
45. Cobra is a patient who suffered an inferior
septal wall myocardial infarction. Which of the D. “If the resuscitation efforts fail, the
following complications would you suspect in health-care provider will ask you if you want to
Cobra when you note on your assessment a terminate resuscitation efforts.”

jugular venous distention and ascites?


47. An apartment fire broke out near the
hospital. The injured victims are sent to the
A. Left-sided heart failure
emergency department of the hospital. Five
families of the injured patients arrived in the ED
B. Pulmonic valve malfunction
subsequently to inquire about the health status
C. Right-sided heart failure of their family members. Which of the following
is your best action?
D. Ruptured septum
A. Take the families to the triage area so they A. Triage the clients and send them to the
can be with their loved ones appropriate areas.

B. Ask the families to wait in the waiting area B. Thoroughly wash the clients with soap and
until information is available water and then rinse.

C. Ensure that there is a designated area for C. Remove the clients’ clothing and have
family staffed by available social workers or them shower.
clergy
D. Assume the clients have been
D. Direct families to a lounge where a decontaminated at the plant.
receptionist will be keeping families informed
50. You are attending a seminar on codes. After
48. Patient Aokiji is a male client that presented the session, you recall that which of the
themself in the emergency department after following interventions is the most important for
vomiting a “large” amount of bright red blood. you to implement when participating in a code?
Which of the following actions should you
implement first?
A. Elevate the arm after administering
medication.
A. Start an intravenous line with an
18-gauge needle. B. Maintain sterile technique throughout the
code.
B. Have the UAP take the client’s vital signs.
C. Treat the client’s signs/symptoms; do not
C. Ask the client to provide a stool specimen for treat the monitor.
blood.
D. Provide accurate documentation of what
D. Send the client to radiology for an abdominal happened during the code.
CT scan.
Situation: Ms. Hange is the charge nurse of a
49. You are working as a triage nurse in a large medical unit. She is responsible for the
trauma center. The center has been notified of management and supervision of the unit.
an explosion in a nearby major chemical
manufacturing plant. Which of the following
actions should you implement first when the
51. Ms. Hange observes that one of the
injured patients arrive at the emergency
female staff nurses is not performing her duties
department?
very well. Which of the following strategies will
she implement to assist the staff nurse?
A. Discuss with the staff nurse her B. Safe ranges of the drug.
performance and ways she can improve.
C. Client’s socio-economic status.
B. Allow the staff nurse to select own
assignment. D. Client’s reaction to the drug.

C. Assign the staff nurse several clients with 54. Ms. Hange decides what is best for a
various illnesses.
recovering client and acts on the decision
without consulting the client. Ms. Hange is
D. Ask the staff nurse to work as an assistant
charge nurse. applying a moral principle which is
______________.
52. Ms. Hange notes one of the male staff
nurse is frequently absent and his absence has
A. Paternalism
adversely affected the quality of care given to
the clients unit. Which of the following would be B. Beneficence
the BEST approach?
C. Fidelity

A. Talk with the staff nurse regarding the


D. Autonomy
concern and remind him of the standards of
the agency.
Situation: The nurse cares for a female client

B. Write the staff nurse a memorandum who is terminally ill and is experiencing pain.
regarding his absence.

C. Inform the staff nurse that his absence will


be a ground for termination. 55. The nurse prepares a care plan for the
client. The overall goal for the client is
D. Record the absence of the staff nurse in a ________.
log book.

53. Ms. Hange assigns a new staff nurse to The client will:

administer the medications of a client. Which


detail of the client’s drug therapy is the staff A. Achieve control of pain and discomfort.
nurse legally responsible to document? The
B. Receive adequate cerebral oxygenation and
________.
perfusion.

A. Peak concentration time of the drug. C. Be free from infection.


D. Receive life sustaining food and liquids. A. Terminally ill clients require minimum
physical care.
56. The nurse is aware of the document that
expresses a client’s wish for life sustaining B. Health care personnel do not understand
their own feelings about death and dying
treatment in the event of terminal illness or
therefore they avoid caring for terminally clients.
permanent unconsciousness. This document is
the ______; C. Terminally ill clients have the right to die
with dignity.

A. No-code order
D. Terminally ill client’s experiences pain most
of the time.
B. Durable power of attorney
59. The dying client wishes to donate her eyes
C. Living will
after she dies. Which of the following

D. Last will and testament statements is NOT TRUE about organ


donation?
57. The client nears death and requests that
no medication be given that would cause a loss A. Any individual, at least 15 years old of
of consciousness, including pain medication. age and of a sound mind may donate a part
of his body to take the effect after
The nurse would promote the best end-of-life
transplantation needed by the recipient.
care for the client by which of the following?
B. Sharing of human organs or tissues shall be
A. Discuss the request of the dying client with made only through exchange programs duly
family members and respect their wishes. approved by the Department of Health.

B. Comfort is the highest priority in this situation C. The choice to donate an organ must be a
so give medications as ordered. written document.

C. Respect the client’s wishes and withhold D. Laws do not require the consent of a family
pain medications and other medications members to retrieve organs if the donor has
ordered. expressed his last wish to donate.

D. Be compassionate and give half of dose of Situation: The nurse in the emergency
the medication ordered. department admits a 45 year old female for
vomiting blood. According to a family member
58. Which of the following statement is TRUE
who accompanied the client, the client had a
about terminally ill clients?
gastric ulcer for several years. The nurse
assesses that the client is in shock. B. Anaphylactic reaction

C. Altered level of consciousness

60. Which of the following assessment findings D. Pain


indicate hypovolemic shock?
63. The nurse understands that the best
indication that fluid replacement for the client in
A. Systolic blood pressure is less than 90
mmHg. hypovolemic shock is adequate is when the
___________:
B. Pupils are unequally dilated.

A. Systolic blood pressure is above 110 mmHg.


C. Respiratory rate is more than 30 breaths per
minute.
B. Diastolic blood pressure is above 90 mmHg.

D. Pulse is less than 60 beats per minute


C. Urine output of 20- 30 mL/Hour.

61. In the early stages of shock, the nurse


D. Urine output is greater than 30 mL/Hour.
expects the result of arterial blood gas (ABG)
analysis to indicate which of the following 64. The physician schedules the client for
conditions ____________: surgery within six hours. The nurse minimize
anxiety of the client by answering the client’s
A. Respiratory alkalosis questions regarding the surgery in calm
manner, keeps the client warm, advise the
B. Respiratory acidosis
client to be on bed rest and dims the lights in

C. Metabolic alkalosis the room. The reason for these interventions is


to ________:
D. Metabolic acidosis
A. Increase comfort of the client and her
62. The physician orders intravenous infusion family.
of packed red blood cells and normal saline
solutions. The nurse assesses the client for B. Minimize oxygen consumption.
which of the following _____________:
C. Prevent infection.

A. Hypovolemia D. Stabilize fluid and electrolyte balance.


Situation: A 60 year old male is admitted to the 67. The nurse knows that hormone therapy is
oncology unit. According to the client, he felt a the mode of treatment for a client with prostate
cancer. The goal of this form of treatment is to
growth during a routine digital prostate
______:
examination. He complains of pain on urination
and frequent urination. A. Limit the amount of circulating
androgens.

B. Increase prostaglandin level.


65. The nurse understands that the function of
the prostate gland is primarily to ______; C. Increase the amount of circulating
androgens.

A. Regulate the acidity and alkalinity


D. Increase testosterone level.
environment for proper sperm development.

68. The nurse writes a nursing diagnosis of


B. Produce a secretion that aids the
nourishment and passage of sperm. Fear and Anxiety secondary to the diagnosis of
prostate cancer. Which of the following
C. Secrete a hormone that stimulates the interventions would be BEST for the nurse?
production and maturation of sperm.

D. Store undeveloped sperm before ejaculation. A. Encourage the client to keep his feelings to
himself so his family will not be affected.

66. The nurse analyzes the laboratory values


B. Establish a nurse patient therapeutic
and notes that the serum phosphate level is
relationship.
elevated. This finding indicates which of the
following: C. Advise the client to have a positive outlook
relationship.

A. It confirms the diagnosis of prostate cancer.


D. Provide spiritual support to the client.

B. The progression or regression of prostate


Situation: Nurse Petra works in the oncology
cancer.
unit. She takes care of cancer patients in pain.
C. The likelihood of metastasis to the bones. She is aware that cancer pain management is
one of her responsibilities.
D. There are complications associated with
cancer.
69. Nurse Petra plans care for a cancer client B. Follow the physician’s order for the first 24
experiencing pain. She is aware that an hours.

important principle of using medication to


C. Reassess the client every 8 hours for drug
manage pain is to:= effectiveness.

A. Individualize the medication therapy to the D. Seek a new order after 2 doses that do
client. not achieve a tolerable level of pain relief.

B. Provide the medication as soon as the 72. One of the clients experiences severe,
client requests for it. intractable pain and complains that the pain
medication is not working for him. Which of the
C. Discontinue the medications periodically to
following actions is MOST appropriate for Nurse
discourage the development of drug tolerance.
Petra?
D. Avoid giving client addictive medications.
A. Suggest to the client to try deep breathing to
70. Nurse Petra collaborates with the physician cope with the pain.
in the development of a drug regimen for the
clients. Which of the following medications B. Explore the nature of the pain and
encourage the client to perceive it in a different
should be avoided in the treatment of cancer
way.
pain?
C. Support the client emotionally and tell him he
will receive the next dose of medication as soon
A. Morphine
as possible.

B. Acetaminophen (Tylenol)
D. Refer the client to the attending physician
immediately and report that the pain
C. Meperidine (Demerol) medication is not providing adequate pain
relief.
D. Hydrocodone
73. Nurse Petra assesses a client complaining
71. When titrating a drug for the client in pain,
of acute pain. The MOST appropriate nursing
which of the following actions is MOST
assessment would include which of the
appropriate?
following?

A. Ask the physician to include a medication


A. The nurses’ impression of clients’ pain.
order for breakthrough pain.

B. The clients’ pain rating.


C. Nonverbal cues from the client. c. master patient index file

D. Pain relief after appropriate nursing d. hospital library record file


interventions.
76. Ms. Helen is aware that when a client is
Situation: Ms. Helen is a nurse supervisor of readmitted to a hospital, the client’s file is
three departments in hospital X. She attends an retrieved from the__________.
orientation seminar on hospital records
management.
a. Physician’s file

b. Civil service file

74. Ms. Helen understands that good client care


c. Master patient index file
relies on good record keeping. Which of the
following is NOT a purpose of hospital record d. Hospital library record file
keeping?
77. Ms. Helen is aware that when a client is

a. Records provide evidence of a hospital’s discharged or dies, the following details should
accountability. be entered in the client’s record which is
the_______________.
b. Records are a key source of data for medical
research or statistical reports.
a. Final diagnosis
c. Records provide data on health information
system. b. Outcomes classification

d. Records provide personal information on c. Educational attainment


the physicians and nurses caring for the
clients. d. Religion

75. Ms. Helen is aware that when a client is 78. The following statements are true about
readmitted to a hospital, the client’s file is patients and hospital records EXCEPT:
retrieved from the _______________.
a. Confidential records must be protected
a. physician’s file against loss, damage , unauthorized access,
modification and disclosure

b. civil service file


b. Patients have the right to confidential problems. Which step of the nursing process is
treatment of information they provide to health in effect in this situation?
professional

c. Health records are the property of a. nursing diagnosis


community where the patient is treated
b. assessment
d. Hospital records maybe released without the
patient’s consent when required in investigation c. evaluation
for serious criminal offenses
d. planning
Situation: Ms. Mika is a director of the critical
care unit of hospital x. She utilizes the nursing 81. Ms. Mika consults with the attending
process to communicate care to the client. physician and the anesthesiologist. She advises
the primary nurse to proceed with the
preparations and to remain alert for any
adverse symptoms. Which step of the nursing
79. She is called to the bedside of a client who
process is this?
is scheduled to have laparoscopic
cholecystectomy. The client’s pulse is slightly
irregular. Ms. Mika confers with the primary a. Assessment

nurse regarding the client’s condition, which


b. nursing diagnosis
step of the nursing process is Ms. Mika
applying?= c. planning

a. Implementation d. evaluation

b. Evaluation 82. Ms. Mika confers with the client’s primary


nurse the following morning. Together they
c. Planning determine that the client is ready for surgery.
This step of the nursing process is:
d. Assessment

80. Ms. Mika calls for a conference with the a. evaluation

staff members who are attending to the client.


b. planning
They decide to obtain a 12-lead ECG for a
more definitive picture. They conclude that the c. nursing diagnosis
client has no serious cardiac or pulmonary
d. assessment a. “Wash the dishes in hot soap as you
usually do.”
83. Ms. Mika applies the human relations
approach in this situation. She is aware that the b. “Let the dishes soak in hot water overnight
before washing.”
key to productivity is _________________.

c. “You should boil the client’s dishes for 30


a. the degree of independence allowed minutes after use.”

b. meeting the objectives of the critical care unit d. “have the client eat from paper plates so they
can be discharged.”
c. Firm control of the situation
86. During an AIDS education class a client
d. the behavior of people under direction states, “Vaseline works great when I use
condoms.” Which conclusions about the client’s
Situation: A mother with the diagnosis of AIDS knowledge of condom use can the nurse draw
states that she has been caring for her baby this statement?
even though she has not been feeling well.

a. an understanding of safer sex

84. What important information should the b. an ability to assume self-responsibility

nurse determine?
c. ignorance concerning correct condom
use
a. is she has kissed the baby
d. ignorance concerning the transmission of
b. if the baby is breastfeeding HIV

c. when the baby last received antibiotics 87. The client with AIDS is experiencing
nausea and vomiting. The Nurse would make
d. how long she has been caring for the baby which of the following dietary alterations for this
client to enhance nutritional intake?
85. The nurse is planning to provide discharge
teaching to the family of a client with AIDS.
a. Avoid dairy products and red meat
Which statement should the nurse include in
the teaching plan? b. Plan large nutritious meals

c. Add spices to food to enhance flavour


d. Serve foods while they are warm 90. Which finding best suggests that nursing
interventions for a client with an excess fluid
88. The Physician orders a Paracentesis. How volume have been effective?
should the nurse instruct the client to prepare
for the radiograph?
a. clear breath sounds

a. void before the procedure b. positive pedal pulses

b. a laxative the evening before the procedure c. normal potassium level

c. nothing by mouth for 8 hours before the d. increased urine specific gravity
procedure
91. The nurse understands that a client with
d. a low soapsuds enema the morning of the albuminuria has edema because of:
procedure

Situation: In a Medical ward there are clients a. fall in tissue hydrostatic pressure
with potential or actual disorders of fluids and
b. rise in plasma hydrostatic pressure
electrolytes disturbance and homeostatic
mechanisms. c. rise in tissue colloid osmotic pressure

d. fall in plasma colloid oncotic pressure

89. The nurse is caring for a client with chronic


92. When the nurse uses the clamp on the
kidney failure. The nurse understands that
administration set to manually adjust the flow of
ammonia is normally exerted by the kidney to
IV fluid into a client by gravity, what change in
help maintain:
energy takes place?

a. osmotic pressure of the blood


a. potential energy is converted to kinetic
energy
b. acid-base balance of the body
b. kinetic energy is converted to potential
c. low bacterial level in the urine energy

d. normal red blood cell production c. chemical energy is converted to kinetic


energy
d. potential energy is converted to chemical personnel and staff in the unit regarding trends
energy and treatment for cancer. Nurse Gab read an
article entitled “Understanding Colorectal
93. The client with which condition has an Cancer” which was recently published in a
national newspaper.
increased risk for developing Hyperkalemia?

a. Crohn’s disease 95. According to the Philippine Cancer Facts


and Estimates for 2010, one of the most
b. Cushing’s syndrome
common cancer among men is colorectal
cancer. It ranks ______ among all the diseases:
c. Chronic heart failure

d. End-stage renal disease a. First

Situation: The nurse assists in the care of a 15 b. Fourth


year old female experiencing anaphylaxis due
to insect bite by honeybees. c. Second

d. Third

94. Upon assessment, the nurse observes the Situation: A 21 year old male is admitted to the
client reacting to the insect bites. The following burn unit of x hospital. He sustained burns on
are common reactions to an insect sting the chest, abdomen, right arm and right leg.
EXCEPT:
96. The nurse assigned to his care anticipates
that the client would be particularly susceptible
a. Swelling to which of the following fluid and electrolyte
imbalances during the emergent phase of burn
b. Redness
case.

c. Appearance of lesions
a. Metabolic acidosis
d. Pain
b. Hypernatremia
Situation: Nurse Gab is a staff nurse in the
oncology unit of a tertiary hospital. An activity in c. Hypokalemia
the unit for continuing professional development
d. Metabolic alkalosis
is to disseminate information among the
97. The nurse assesses the client for fluid 100. The client receives total parenteral
shifting. During the emergent phase of a burn nutrition (TPN). The nurse understands this
injury, shifts occur due to fluid moving from therapy will help the client__________.
the_______________.
a. Provide adequate nutrition
a. Extracellular to intracellular space.
b. Ensure adequate caloric and protein
b. Intracellular to extracellular space. intake

c. Vascular to interstitial space. c. Correct water and electrolyte imbalances

d. Interstitial to vascular space d. Allow the gastrointestinal tract to rest

98. The nurse understands that the fluid shift RECALLS 7 - NP5

results from an increase in the_____________.:


Situation: You are a mental health nurse
dealing with patients currently experiencing a
a. Total volume of intravascular plasma crisis. You use your knowledge on crisis
management to help these patients. The
b. Total volume of circulating whole blood
following questions apply.

c. Permeability of capillary walls

d. Permeability of the kidney tubules


1. In the rape crisis center, a woman is being
seen a few days after she was raped. She
99. The client receives fluid resuscitation
reports that she has not had any appetite, she
therapy. The nurse adjusts the infusion rate by
is experiencing anxiety and depression, and
evaluating the client’s __________:
that she has been having nightmares. You as
the nurse make an assessment on the woman
a. Hourly urine output
to determine if it is appropriate for her to be
b. Daily body weight admitted to the hospital. Which of the following
priority questions should the nurse ask the
c. Hourly urine specific gravity woman?

d. Hourly body temperature


A. The client’s reaction to the event,
including any suicidal thoughts.
B. The client’s perceptions of her current skills C. “You’re only 40 years old and not too old to
for coping with the event. have a baby.”

C. The availability of the client’s personal D. “I see you’re upset. Take some deep
support systems. breaths to relax a little.”

D. The effect of the event on other aspects of 4. You were doing some charts when a client
the client’s life. comes to the crisis center in a very distressed
state. He tells you that he just cannot get over
2. An anxious, sobbing 19 year old is brought
being fired from his job last week. He says that
to the crisis shelter for an interview. She says, “I
he already asked for help and talked to friends.
think I am pregnant but I don’t know what to
He says, “I’ve tried everything to get through
do!” Which of the following nursing interventions
this, but nothing is working. Please, help me!”
is most appropriate for her situation at this
Which of the following should you, as the nurse,
time?
use as the initial crisis intervention strategy?

A. Ask the client about the type of things that


she had thought of doing. A. Referral for counseling.

B. Give the client some ideas about what to B. Support system assessment.
expect to happen next.
C. Emotion management.
C. Recommend a pregnancy test after
acknowledging the client’s distress. D. Unemployment assistance.

D. Question the client about her feelings and 5. Getting the client’s significant others involved
possible parental reactions. in helping with the immediate crisis as soon as
possible is one of the major roles in crisis
3. You’re the nurse on duty when you saw an
intervention. You as the nurse determine that
anxious 41-year-old client say that she would
the support persons are prepared to help when
“rather die than be pregnant.” Which of the
they verbalize which of the following?
following responses by the nurse is most
helpful?
A. The name and phone number of the client’s
physician.
A. “Try not to worry until after the pregnancy
test.” B. Emergency resources and when to use
them.
B. “You know, pregnancy is a normal event.”
C. The coping strategies they are using. C. Decreasing interaction with peers to
conserve energy.
D. Long-term solutions they plan to tell the
client to use. D. Gaining an intellectual understanding of the
illness.
Situation: You are a nurse tasked to work with
patients coping with their illnesses. 8. The client hospitalized for diagnosis and
treatment of atrial fibrillation states to the nurse,
“Please hand me the telephone. I need to check
on my stocks and bonds.” Which of the
6. A client was diagnosed with an acute cardiac
following responses by the nurse is most
illness. The nurse should determine that the
therapeutic?
client lacks understanding of her illness and her
ability to make changes in her lifestyle when
A. “You will get more upset if you make that
they verbalize which of the following
call.”
statements?
B. “You have atrial fi brillations. Let’s talk
about what that means.”
A. “I already have my airline ticket, so I
won’t miss my meeting tomorrow.”
C. “You really don’t care about the fact that
you’re sick, do you?”
B. “These relaxation tapes sound okay; I’ll see if
they help me.”
D. “Do you realize you have a life-threatening
condition?”
C. “No more working 10 hours a day for me
unless it’s an emergency.”
9. The colostomy club made arrangements to
D. “I talked with my husband yesterday about meet with a client who will undergo a bowel
working on a new budget together.” surgery. Which of the following is accomplished
when a representative of the colostomy club
7. You just admitted a 19-year-old client who
visits the client preoperatively?
was recently diagnosed with leukemia. What is
the most appropriate short term goal for the
A. Letting the client know that he has resources
nurse and the client to establish?
in the community to help him.

A. Accepting his death as imminent. B. Providing support for the physician’s plan of
therapy for the client.
B. Expressing his angry feelings to the
nurse.
C. Providing the client with support and A. Telling her to distract himself with games and
realistic information on the colostomy. television

D. Convincing the client that he will not be B. Reassure her that she will come through the
disfigured and can lead a full life. surgery without incident

10. One of your patients in the ward directs C. Explaining to her what happens before
profanities at you, the nurse, then abruptly and after surgery

hangs his head and pleads to you, “Please


D. Asking the surgeon to refer her to a
forgive me. Something came over me. Ugh, psychiatrist who can work with her to diminish
why do I say those things?” As a her anxiety
knowledgeable nurse, you interpret this as
which of the following? 12. You are discussing the concept of anxiety to
the student nurses in your unit. You explain that
anxiety occurs in degrees, from a level that
A. Neologism
stimulates productive problem solving to a level
B. Confabulation that is severely debilitating. The students
respond correctly when you ask that at a mild,
C. Flight of ideas productive level of anxiety, one will expect to
see which of the following cognitive
D. Emotional lability
characteristics of mild anxiety?

Situation: You are a nurse tasked to care for


patients experiencing stress and anxiety. You A. Slight muscle tension.
are to apply the nursing concepts you’ve
B. Occasional irritability.
learned about this topic to effectively care for
these patients.
C. Accurate perceptions.

D. Loss of contact with reality

11. You are a nurse tasked to care for patients


13. You followed up a question to the student
experiencing stress and anxiety. You are to
nurses. They answered you correctly when they
apply the nursing concepts you’ve learned
stated that as a client’s anxiety level increases
about this topic to effectively care for these
to a debilitating degree, they would expect
patients.
which of the following psychomotor behavior
indicating the panic level of anxiety:
A. Suicide attempts or violence. C. “May I arrange for some part-time help for
you?”
B. Desperation and rage.
D. “Being a full-time caregiver must be very
C. Disorganized reasoning. stressful, isn’t it?”

D. Loss of contact with reality. Situation: You are a nurse tasked to care for
patients with schizophrenia. You use your
14. You admitted a patient dealing with personal knowledge on this concept to effectively and
issues and painful feelings. Which of the safely care for your patients.
following is a crucial goal of therapeutic
communication when helping this client?

16. You are caring for a patient diagnosed with


A. Communicating empathy through gentle
paranoid schizophrenia. The patient reports
touch
hearing a voice saying “Do not remove your cap
B. Conveying client respect and acceptance or they will be able to read your mind.” Which of
even if not all of the client’s behaviors are the following responses is the most therapeutic
tolerated
for this patient?

C. Mutual sharing of information, spontaneity,


emotions, and intimacy A. “Who are ‘they’?”

D. Guaranteeing total confidentiality and B. “Why would someone want to read your
anonymity for the client mind?”

15. You are doing a follow up visit to the home C. “I do not believe that anyone can read
of a client diagnosed with Alzheimer’s disease. another’s mind.”

You are assessing the stress level of the


D. “It must be very frightening to believe
patient’s spouse, the primary caregiver. Which that someone can read your mind.”
of the following questions is most appropriate
for assessing the spouse’s level of stress? 17. A patient diagnosed with a history of
paranoid schizophrenia and chronic alcohol

A. “So, what is a typical day like for you?” abuse was admitted to your unit. The patient
has been taking Olanzapine for 14 days and
B. “What do you do to relieve stress for has not consumed alcohol in the last 5 days.
yourself?”
They report shaky hands and trouble sleeping
because of frequent nightmares. The patient 19. A patient diagnosed with paranoid
verbalized their concern that olanzapine may be schizophrenia was admitted to your unit. You
include the nursing diagnosis of Disturbed
causing these problems. Which of the following
thought processes secondary to paranoia in the
is your most therapeutic response to this patient’s care plan. Which of the following
patient? approaches is most appropriate for this patient?

A. Avoid laughing or whispering in front of


A. “These are not typical side effects for that
the client.
drug.”

B. Begin to identify social support in the


B. “Just ignore the symptoms. They will go
community.
away in just a few days.”

C. Encourage the client to interact with others


C. “These symptoms are more likely a result
on the unit.
of not drinking alcohol for 5 days.”

D. Have the client sign a written release of


D. “It is possible, since this medication is
information form.
contraindicated in those who abuse alcohol.”

20. The mother of a client diagnosed with


18. A patient with a history of violent command
paranoid schizophrenia visiting her son 2 days
hallucinations was observed to be mumbling
after his admission to the psychiatric unit
erratically while making threatening gestures
approaches a nurse and states, “He is still
directed toward a particular staff member.
talking about how the government is controlling
Which of the following interventions is most
his thoughts.” What is the most accurate
appropriate when caring for patients with violent
nursing appraisal of the mother’s statement?
command hallucinations?

A. The mother’s expectations of her son are


A. Ask the client to explain the cause of
realistic.
anger.

B. The mother’s concern is reasonable.


B. Place the client in seclusion to help
de-escalate anger.
C. The mother should request a medication
adjustment.
C. Inform the client of pending restraint if
behavior does not subside.
D. The mother requires further education
regarding the client’s diagnosis.
D. Observe the client for signs of escalating
agitation.
Situation: You are tasked to care for Zoro, a 23. Which cognitive symptom would you expect
patient newly diagnosed with obsessive to assess in Zoro who has obsessive
compulsive disorder. You use your knowledge compulsive disorder?
to effectively and safely care for the patient.
A. Compulsive behaviors that occupy more than
4 hours per day.

21. Zoro is utilizing a defense mechanism B. Excessive worrying about germs and
commonly used by patients with obsessive illness.
compulsive disorder. Which of the following
defense mechanisms is this? C. Comorbid abuse of alcohol to decrease
anxiety.

A. Suppression. D. Excessive sweating and an increase in blood


pressure and pulse.
B. Repression.
24. Zoro is leaving his home for the first time in
C. Undoing. a year. He arrived in the unit wearing a surgical
mask and white gloves. He states, “The germs
D. Denial.
in here are going to kill me”. Which correctly

22. You start your assessment on Zoro. Which written nursing diagnosis addresses Zoro’s

behavioral symptom would you expect to problem?

assess in this patient?


A. Social isolation R/T fear of germs AEB
continually refusing to leave the home.
A. The client uses excessive hand washing
to relieve anxiety.
B. Fear of germs R/T obsessive-compulsive
disorder.
B. The client rates anxiety at 8/10.

C. Ineffective coping AEB dysfunctional


C. The client uses breathing techniques to isolation R/T unrealistic fear of germs.
decrease anxiety.

D. Anxiety R/T the inability to leave home,


D. The client exhibits diaphoresis and resulting in dysfunctional fear of germs.
tachycardia.

25. Zoro has been in your care in the


psychiatric unit for 4 days now for the treatment
of their OCD. Which outcome takes priority for D. Improved sleep
the patient at this time?
27. As a knowledgeable nurse, you know that
body image is the subjective view an individual
A. The client will use a thought-stopping
technique to eliminate obsessive and/or has about his or her physical appearance
compulsive behaviors. including body shape, size, weight, and
proportions. Which of the following conditions
B. The client will stop obsessive and/or
would put a patient at risk for disturbed body
compulsive behaviors in order to focus on
activities of daily living. image?

C. The client will seek assistance from the staff A. Urinary tract infection
to decrease obsessive and/or compulsive
behaviors.
B. Hyperlipidemia

D. The client will use one relaxation


C. Rheumatoid arthritis
technique to decrease obsessive and/or
compulsive behaviors.
D. High blood pressure
Situation: You apply your knowledge on
28. You are a preoperative nurse preparing a
concepts of psychosocial health to patients
client for an upcoming surgery. While you’re
assigned to you in the ward.
preparing this patient, you inform them of what
they can expect after surgery and how their
pain will be controlled postoperatively. Which of
26. You are attending a seminar regarding the following stress management techniques is
coping skills. You were asked about the being utilized in this scenario?
beneficial effects of humor. You respond to the
question appropriately based on which of the
A. Relaxation
following documented beneficial effects of
humor? B. Guided imagery

C. Progressive muscle relaxation


A. Lessened depression

D. Anticipatory guidance
B. Increased relaxation

29. An elderly patient you’re caring for is about


C. Reduced aggression
to be discharged. Which of the following
statements, if made by the patient, would D. I, II, III, IV
indicate that they lack a support system at
home? Situation: You are caring for various patients
with substance abuse disorder of
methamphetamines. You utilize your knowledge
A. “My sister and her husband are taking me
home today.” to help care for these patients.

B. “My church members have been sending


cards and letters while I have been in the
hospital.” 31. You are assessing a patient diagnosed with
substance abuse disorder. They stated, “My
C. “I am not sure how I am going to get to wife causes me to abuse methamphetamines.
the grocery store after I get home.”
She uses methamphetamine and she also

D. “My neighbor is retired. We visit and have expects me to.” As a knowledgeable nurse, you
our meals together every day.” know that the patient is using which of the
following defense mechanisms?
30. You are to assess a newly admitted patient
regarding their health care practices. As a
A. Rationalization.
culturally competent nurse, which of the
following factors would you include in your B. Denial.
assessment?
C. Minimization.

I. Health-seeking behaviors
D. Projection.

II. Responsibility for health care 32. The mother of one of your patients who are
newly admitted to the mental health unit
III. Folklore practices expresses her concern that his son may be
using methamphetamine. Which physical

IV. Barriers to health care examination findings are consistent with


methamphetamine abuse by the client?

A. I
A. Hypotension and bradycardia
B. III
B. Bruises and scrapes on the extremities
C. I, II, IV
C. Constricted pupils and fatigue D. “Good point. You probably work long hours
while you are on meth.”
D. Anorexia and recent weight loss

33. One of the patients you’re caring for in the


unit is in methamphetamine withdrawal. When 35. One of your patients regularly uses
caring for this patient, the most appropriate projection to protect themselves against the
intervention by the nurse should be to? negative realities resulting from their
methamphetamine use. Which of the following

A. Administer sedatives routinely to prevent statements will the nurse most likely document
seizures. when the patient uses projection as a coping
mechanism?
B. Allow the client to sleep and eat as
desired.
A. “My dad and I don’t get along because he
C. Administer antipsychotic medications to thinks that I’m a failure.”
manage hallucinations.
B. “I can’t go back to work. I’d be so
D. Encourage involvement in the treatment embarrassed for anyone to find out I’ve been in
milieu. treatment.”

34. You are assessing one of the patients in C. “I’m not giving up alcohol, just the
methamphetamine. I never had a problem with
your unit who abuses methamphetamine. The alcohol.”
patient appears not to be willing to give up the
usage of the drug, as evidenced by their D. “Everything will be all right again if I can just
statement, “I do not plan to quit meth. I can stop using drugs.”

work for days when I am high.” Which of the


Situation: You are caring for Robin, a patient
following is your best response to the patient’s
diagnosed with Multiple Sclerosis. The following
statement?
questions apply.

A. “You’ll exhaust yourself doing that.”

B. “You can’t see the real problem yet because 36. Robin is scheduled for a magnetic
you are in denial.” resonance imaging (MRI) scan of the head.
Which of the following information should you
C. “You think using drugs helps you?”
relay to Robin about the test?
A. The client will have wires attached to the B. She wants to discuss if she should tell her
scalp and lights will flash off and on. fiancé.

B. The machine will be loud and the client C. She tells the nurse life is not worth living
must not move the head during the test. anymore.

C. The client will drink a contrast medium 30 D. She needs the flu and pneumonia vaccines.
minutes to one (1) hour before the test.
39. Robin stated that she has been
D. The test will be repeated at intervals during a investigating alternative therapies to treat her
five (5)- to six (6)-hour period.
disease. Which of the following interventions is

37. Robin stated her frustration regarding her most appropriate?

recent diagnosis of MS. She states, “I do not


understand how I got this disease. Is it A. Encourage the therapy if it is not
contraindicated by the medical regimen.
genetic?” On which statement should you base
your response?
B. Tell the client only the health-care provider
should discuss this with him.
A. Genetics may play a role in susceptibility to
MS, but the disease may be caused by a virus. C. Ask how his significant other feels about this
deviation from the medical regimen.
B. There is no evidence suggesting there is any
chromosomal involvement in developing MS. D. Suggest the client research an
investigational therapy instead.
C. Multiple sclerosis is caused by a recessive
gene, so both parents had to have the gene for 40. You enter Robin’s room after her diagnosis
the client to get MS. of acute exacerbation of MS. You find her
crying. Which of the following statements is the
D. Multiple sclerosis is caused by an autosomal
most therapeutic response you can make as
dominant gene on the Y chromosome, so only
fathers can pass it on. her nurse?

38. Which of the following issues presented by


A. “Why are you crying? The medication will
Robin is of most importance to you at this time help the disease.”
as her primary nurse?
B. “You seem upset. I will sit down and we
can talk for awhile.”
A. She refuses to have a gastrostomy feeding.
C. “Multiple sclerosis is a disease that has good B. Tell the client not to eat anything for eight (8)
times and bad times.” hours prior to the procedure.

D. “I will have the chaplain come and stay with C. Instruct the client to stay awake for 24
you for a while.” hours prior to the EEG.

Situation: You are caring for patient Luffy who D. Explain to the client that there will be some
has seizures. You apply your knowledge on discomfort during the procedure.

concepts of seizures to better assess,


43. Luffy just had a 3 minute seizure. He has
diagnose, plan, and evaluate their condition.
no apparent injuries, is oriented to name, place,
and time but he is very lethargic and just wants
to sleep. Which of the following interventions
41. Luffy is sitting in the chair when suddenly, should you implement?
his entire body went rigid with his arms and legs
contracting and relaxing. He is not aware of A. Perform a complete neurological
what’s going on and is making guttural sounds. assessment.
Which of the following actions should you
implement first? B. Awaken the client every 30 minutes.

C. Turn the client to the side and allow the


A. Push aside any furniture. client to sleep.

B. Place the client on his side. D. Interview the client to find out what caused
the seizure
C. Assess the client’s vital signs.
44. Which statement by Luffy indicates that he
D. Ease the client to the floor. understands factors that may precipitate his
seizure activity?
42. Luffy is scheduled for an
electroencephalogram (EEG) to help diagnose
A. “It is all right for me to drink coffee for
a seizure disorder. Which of the following
breakfast.”
preprocedure teaching should you implement?
B. “My menstrual cycle will not affect my seizure
disorder.”
A. Tell the client to take any routine antiseizure
medication prior to the EEG.
C. “I am going to take a class in stress
management.”
D. “I should wear dark glasses when I am out in 47. You are conducting an interview with the
the sun.” surgical patient in the holding area. Which of
the following information should you report to
45. Luffy is prescribed the anticonvulsant the anesthesiologist? Select all that apply.

phenytoin (Dilantin) for his seizure disorder.


I. The client has loose, decayed teeth.
Which statement indicates that Luffy
understands the discharge teaching regarding
II. The client is experiencing anxiety.
this medication?

III. The client smokes two (2) packs of


A. “I will brush my teeth after every meal.”
cigarettes a day.
B. “I will check my Dilantin level daily.”
IV. The client has had a chest x-ray which
C. “My urine will turn orange while on Dilantin.”
does not show infiltrates.

D. “I won’t have any seizures while on this


medication.” V. The client reports using herbs.

Situation: You are a new nurse assigned in the


A. I, II, III, IV
operating room. You will apply your knowledge
on perioperative nursing to effectively and B. II, III
safely handle patients in this area.
C. I, III, V

D. III, V
46. You are preparing your patient for an
upcoming surgery. Which of the following 48. The circulating nurse intervenes when she
interventions should you implement first? notices which of the following violations of
surgical asepsis?

A. Check the permit for the spouse’s signature.


A. Surgical supplies were cleaned and sterilized
B. Take and document intake and output. prior to the case.

C. Administer the sedative. B. The circulating nurse is wearing a long


sleeve sterile gown.
D. Complete the preoperative checklist.
C. Masks covering the mouth and nose are
being worn by the surgical team.
D. The scrub nurse setting up the sterile c. Antibiotics do not easily penetrate the
field is wearing artificial nails. infected bony structure of the mastoid

49. The following statements are not an d. Culture has to be done to identify which
expected outcome for the postoperative client antibiotic is most effective for the treatment of
Mastoiditis
who had a general anesthesia, except?

Situation: You are a staff nurse in a


A. The client will be able to sit in the chair for 30 government hospital being transferred to the
minutes.
Psychiatric Unit. You were required to equip
yourself by attending the enhancement program
B. The client will have a pulse oximetry
reading of 97% on room air. on Crisis Intervention. To assess your
knowledge and skills on the subject you were
C. The client will have a urine output of 30 mL given a pre-test.
per hour.

D. The client will be able to distinguish sharp


from dull sensations. 52. A crisis that is acute but temporary and due
to an external source is__________.
50. Which of the following problems should you
identify as the priority for a patient who one day
a. Developmental
postoperative?

b. Transitional
A. Potential for hemorrhaging.
c. Traumatic
B. Potential for injury.
d. Dispositional
C. Potential for fluid volume excess.
53. The MAIN objective of crisis intervention is
D. Potential for infection. to_____________

51. Antibiotics have limited use in the actual


a. Make the person realize his/her mistakes
treatment of Mastoiditis because________.

b. Ensure patient’s safety


a. Tissue destruction is extensive
c. Return the person to the root of the crisis
b. It is a long-term treatment to identify the cause
d. Eliminate the stressor a. In a child, pull pinna upward and backward.

54. Which of the following is NOT an b. Let the ear drops fall on the middle space of
assumption in the concept of crisis? the canal.

c. Lie on the unaffected side to facilitate


a. Crisis is acute and resolved within a short absorption.
period of time
d. Position unaffected ear uppermost.
b. All individuals experience a crisis
57. The nurse assists in an ear irrigation. Which
c. Crisis is a growth-retarding factor to the
of the following statements by the nurse is
emotional development of a person
correct?
d. Specific identifiable events precipitate a crisis
a. “Tilt the head towards the unaffected ear.”
55. Which of the following nursing interventions
is the most appropriate for a client who is in the b. “Direct the stream of irrigate at the sides
early state of crisis? of the ear canal.”

c. “After the procedure, lie on the unaffected


a. Encourage client to express feeling and side to allow the irrigate to soften any hardened
emotions related to crisis mass.”

b. Require client to be actively involved in d. “This procedure is allowed for otitis media to
establishing goals clean the canal.”

c. Encourage client to begin the development of 58. What makes children more predisposed to
insight
chronic otitis media?

d. Ask client to evaluate the situation


a. Shorter Eustachian tube
Situation: In the PGH Ear Unit, the staff nurse
is attending to several outpatient clients seeking b. Horizontal orientation of the ear canal

follow-up care.
c. Primary diaphragmatic breathing

d. Both A and B
56. In administering ear drops, the nurse
observes which of the following principles?
59. The Psychiatrist orders “Restraints PRN” for a. Atrium
a client who has a history of violent behavior.
Nurse Poppy should: b. Ventricles

A. Utilize the restraint order if the client begins c. Purkinje fibers


to act-out
d. Interventricular septum
B. Ask the psychiatrist to clarify the type of
restraint order
62. Which of the following do not have the
potential of addiction, if consumed frequently
C. Ensure that the entire staff is aware of the
restraint order and in large amounts?

D. Recognize that PRN orders for restraints a. Chocolate-flavored Cola


are unacceptable

b. Apple juice
60. Which of the following is a characteristic
sign of acute otitis media in children? c. Green tea

a. Jumping in pain d. Common cold preparations

b. Ear tugging 63. In the previous situation of the young


professional intoxicated with caffeine, he
c. Painless inflammation suddenly was unable to take any caffeine
source for 24 hours already. The nurse expects
d. Difficulty awakening
to note the following findings, except?

Situation: Addiction disorders are


unnecessarily common in the modern lifestyle a. Headache

of Filipinos, especially with the rise of


b. Difficulty in stimulating
establishments selling products with caffeine.
Because of the various “improvements” in c. Nausea and vomiting
performance, this industry is still unwavering.
d. Muscle pain

64. The nurse suspects caffeine intoxication in


61. Caffeine greatly affects which part of the
a young professional if he notes which finding?
heart, as reflected in an ECG?
a. Decreased flow of thought and speech c. 2/1

b. Psychomotor agitation d. 1/2

c. Urinary retention 67. A student was not able to read the letters in
the 20/20 level. How should the nurse proceed
d. Pale face with the visual assessment?

65. The following are the reasons why many


a. Document this finding as visual impairment.
people abuse caffeine. Choose the exception.

b. Allow the student to come nearer at a


a. Relieve fatigue distance of 10 ft.

b. Increase mental alertness c. Ask the student to squint, and try reading
the level again.
c. Both A and B
d. Remind the student to avoid guessing at
d. Neither A nor B letters to have an accurate finding.

Situation: The student nurse is reviewing for 68. A patient is due to undergo tonometry for
his admission exam for a prestigious hospital in confirmation of the diagnosis of glaucoma. The
Taguig City. He is answering questions related nurse advises the patient against which of the
to eye disorders. following, except:

a. Squinting

66. In the clinic, the school health nurse is


b. Breathing through open glottis
conducting a vision screening to incoming
Grade 1 and Grade 4 students. One of the c. Coughing
students was able to read at 10 ft, what a
normal eye sees at 20 feet. She documents this d. Bending at the hips

finding as:
69. The nurse is caring for a client following
enucleation. The nurse notes the presence of
a. 10/20
bright red drainage on the dressing. Which
nursing action is appropriate?
b. 20/10
a. Notify the physician. a. encourage bed rest

b. Document the finding. b. space activities throughout the day

c. Continue to monitor the drainage. c. teach the limitations imposed by the disease

d. Mark the drainage on the dressing and d. have one of the client’s relatives stay at the
monitor for any increase in bleeding. bedside

70. The nurse is performing an admission 72. Which clinical indicator does Nurse Jeremy
assessment on a client with a diagnosis of identify when assessing a client with
detached retina. Which of the following is hemiplegia?
associated with this eye disorder?
a. paresis of both lower extremities
a. Total loss of vision
b. paralysis of one side of the body
b. Pain in the affected eye
c. paralysis of both lower extremities
c. A yellow discoloration of the sclera
d. paresis of upper and lower extremities
d. A sense of a curtain falling across the
field of vision 73. Which statement by a client with Multiple
Sclerosis indicates to Nurse Jeremy that the
Situation: The diverse Neurologic disorders client needs further teaching?
present unique challenges of nursing care. The
Nurse must have a clear understanding of the
a. “I use a straw to drink liquids.”
pathologic processes for appropriate nursing
management. Nurse Jeremy is attending to b. “I will take a hot bath to help relax my
clients in the ward with Multiple Sclerosis. muscles.”

c. “I plan to use an incontinence pad when I go


out.”
71. A recently hospitalized client with Multiple
d. “I may be having a rough time now, but I
Sclerosis is concerned about generalized
hope tomorrow will be better.”
weakness and a fluctuating physical status.
What is the priority nursing intervention for this 74. Mr. Dela Cruz a 48 year old client carpenter
client? admitted after a spinal cord injury and the
Physician indicates that a client is a Paraplegic.
The family asks Nurse Jeremy what this means. irreversible Dementia.
What explanation should the nurse give to the
family?

a. upper extremities are paralyzed 76. The Nurse develops a nursing diagnosis of
self care deficit for an older client with
b. lower extremities are paralyzed Dementia. Which of the following is the most
appropriate goal for this client?
c. one side of the body is paralyzed

d. both lower and upper extremities are a. The client will be admitted to a long care
paralyzed facility to have activities of daily living needs
met
75. Jeremy is excited to be assigned to a Neuro
b. The client will function at the highest level
–Ward after his extensive training. He is
of independence possible
preparing to conduct a Neurologic examination.
What nursing intervention is anticipated for a c. The client will complete all activities of daily
client in the plateau phase of Guillain-Barre living independently within one (1 ) hour time
frame
syndrome?

d. The Nursing staff will attend to all the client’s


a. providing a straw to stimulate the facial activities of daily living needs during the
muscles hospitalization

b. inserting an indwelling catheter to 77. The nurse recognizes that Dementia of the
monitor urinary output
Alzheimer’s type is characterized by:

c. encouraging aerobic exercises to avoid


muscle atrophy a. aggressive acting-out behavior

d. administering antibiotic medication to prevent b. periodic remissions and exacerbations


pneumonia
c. hypoxia of selected areas of brain tissue
Situation: In the Psychiatric ward nurses are
discussing the other factors that caused d. areas of brain destruction called senile
plaques
Alzheimer's disease (AD). And they all agree
that it is a degenerative disease of the brain
78. When attempting to understand the
caused by gradual death and loss of brain cells
behavior of an older adult diagnosed with
resulting to progressive and
Vascular Dementia, the nurse recognizes that last month he has changed from being placid
the client is probably: and easygoing to agitated and aggressive. He
is admitted to a Psychiatric hospital for
a. not capable of using any defense treatment with Psychopharmacology. Which is
mechanisms the priority nursing care while this client is in the
psychiatric facility?
b. using one method of defense for every
situation
a. managing his behavior
c. making exaggerated use of old, familiar
mechanism b. preventing further deterioration

d. attempting to develop new defense c. focusing on the needs of the wife


mechanism to meet the current situation.
d. establishing on the needs of the wife
79. Which of the following nursing intervention
is most helpful in meeting the needs of an older Situation: The ICU nurse assigned to a 60-year
adult hospitalized with the diagnosis of old acutely ill client with Parkinson’s disease
Dementia of the Alzheimer’s type? who was hospitalized frequently. The initial
confinement was due to electrolyte imbalance.

a. providing a nutritious diet high in The following confinement was due to injury
carbohydrates and protein sustained from fall, he became to have
incontinent of stools that further lead to
b. simplifying the environment as much as
development of skin irritation and breakdown.
possible while eliminating the need for
choices Currently he was admitted due to respiratory
infection.
c. developing a consistent nursing plan with
fixed time schedules to provide for emotional
needs
81. The review of literature does not only
d. providing an opportunity for many alternative include published research studies but also
choices in the daily schedule to stimulate
theory. In this case which theory is least related
interest
to the study?
80. A 75-year-old man with the diagnosis of
Dementia has been cared for by his wife for 5 a. Neuman’s system model
years. For the past 2 years he has not spoken
and incontinent of urine and feces. During the b. Lazarus’ theory of stress and coping
c. Nightingale’s environmental theory d. "You need to choose the alternative
therapy that is right for you based on
d. Roy’s theory of adaptation research that supports the intervention."

82. Related literature included case situations 84. While the nurse was able to identify the

similar to the case of the client. The nurse is cases that were studied, it is important to

interested in gaining further knowledge that can understand the phenomenological experience

help the client at risk for fecal incontinence. The of the client. This approach includes the

nurse should use which of the following following except:

methods to strengthen this report?


a. Exploring the idea expressed by the person

a. Historical research method


b. Getting the whole picture of fecal
incontinence and its associated factors
b. Qualitative research method

c. Focusing interview on fecal incontinence


c. Experimental research method

d. Interviewing and using of questionnaire on


d. Quantitative research method
client’s responses to his situation

83. The patient also reports multiple lumbar


85. Which of the following can the nurse use in
muscle strains, thus is also looking at using
protecting the safety of the subjects undergoing
alternative therapies to reduce the pain. The
the research study?
client seeks advice from the nurse as to what
type of alternative therapy would provide the
i. Code for Nurses
best pain relief. How should the nurse respond?

ii. Nightingale’s pledge


a. "I have seen many individuals with your type
of pain be relieved of pain through the use of
acupuncture." iii. Patient’s Bill of Rights

b. "These types of therapies are more than just


iv. Human Rights Guidelines
therapies; they are really a mind over matter
type of event or game."
a. 1, 2, 3, 4
c. "Some of my other clients swear by magnet
therapy to reduce pain as it is very small and b. 1, 3
very easy to use."
c. 1, 2 88. After a brain attack a client remains
unresponsive to sensory stimulation. Nurse
d. 3 only Ymir understands general sensations such as
heat, cold, pain, and touch are registered in the:
Situation: In a Nursing Practice you are directly
involved in conducting a comprehensive
A. frontal lobe
physical assessment especially to older clients
with sensory limitations.
B. parietal lobe

C. occipital lobe

86. The client with head injury is having


D. temporal lobe
problems with several sensory functions. Nurse
Ymir should understand that the structure that 89. Visual Acuity declines with age. Presbyopia
acts as a relay center for sensory impulses is is a progressive decline in:
the:

A. Distinguishing between blues and greens


A. thalamus and among pastel shades

B. cerebellum B. Ability to see in darkness

C. hypothalamus C. The ability of the eyes to accommodate


for close detailed work
D. medulla oblongata
D. Adaptation to abrupt changes from dark
87. When formulating nursing care plans for areas to light areas

older adults, Nurse Ymir should include special


90. The novice nurse who is administering a
measures to accommodate for age-related
beta blocker asks the Senior Staff Nurse about
sensory losses such as:
its effect on the Autonomic Nervous System.
When formulating a response the nurse should
A. difficulty in swallowing
understand which common misconception
about the Autonomic Nervous System?
B. increased sensitivity to heat

C. diminished sensation of pain A. both sympathetic and parasympathetic


impulses continually affect most visceral
D. heightened response to stimuli effectors
B. the autonomic nervous systems is regulated A. Satisfy own needs
by impulses from the hypothalamus and other
parts of the brain B. Identify own needs

C. sympathetic impulses stimulate while C. Satisfy parents’ needs


parasympathetic impulses inhibit the
functioning of any visceral effector
D. Live up to society’s expectations

D. visceral effectors (e.g., cardiac muscle,


93. The nurse encourages a client to join a
smooth muscle, glandular epithelial tissue)
receive impulses only via autonomic neurons self-helping group after being discharged from a
Mental health facility. The purpose of having
Situation: Poppy a Psychiatric Nurse responds people work in a group is to provide:
in a variety setting to different clients with
Personality disorders. A. Support

B. Confrontation

91. The Psychiatrist orders “Restraints PRN” for


C. Psychotherapy
a client who has a history of violent behavior.
Nurse Poppy should: D. Self-awareness

94. As Depression begins to lift, a client is


A. Utilize the restraint order if the client begins
to act-out asked to join a small discussion group that
meets every evening on the unit. The client is
B. Ask the psychiatrist to clarify the type of reluctant to join because, “I have nothing to talk
restraint order
about.” What is the best response by the
nurse?
C. Ensure that the entire staff is aware of the
restraint order
A. “Maybe tomorrow you will feel more like
D. Recognize that PRN orders for restraints talking.”
are unacceptable
B. “Could you start off by talking about your
92. Strict toilet and too early training to a toddler family?”
child will cause problems in personality
development because at this age a child is C. “A person like you has a great deal to offer
the group.”
learning to:
D. “You feel you will not be accepted unless A. Infancy
you have something to say?”
B. School age
95. A client on the Psychiatric unit asks Nurse
Poppy about Psychiatric Advance Directives C. Toddlerhood
(PAD). The nurse explains that these advances
D. Preschool age
directives:

97. When planning to teach about the stages of


A. Make the appointment of a surrogate growth and development, what stage does the
decision maker unnecessary
nurse indicate as basically concerned with role

B. Permit the client to dictate what treatments identification?


will be given during future hospitalization
A. Oral stage
C. Eliminate the need for involuntary
admissions when the client is a threat to self or
B. Genital stage
others

C. Oedipal stage
D. Allow the client, while having the
capacity, to consent or refuse potential
psychiatric treatments in the event of a D. Latency stage
future incapacitating mental health crisis
98. The nurse understands that Freud’s phallic
Situation: The fundamental assumption of stage of psychosexual development, which
theory of life cycle theories is that development compares with Erikson’s psychosocial phase of
occurs in successive stages. The different life initiative versus guilt, is seen best at:
cycle theories try to explain personality
development as well as development of A. adolescent
Psychiatric disorders. The following questions
refer to this situation. B. 6 to 12 years

C. 3 to 51/2 years

96. The nurse understands that problems with D. birth to 1 year


dependence versus independence develop
99. A 3 year old boy was brought to a Pediatric
during the stage of growth and development
clinic for indifferent behavior. About a month
known as:
after their toddler is diagnosed as moderately
retarded, the parents discuss the toddler’s
future, reflecting specifically on plans for their
child’s independent functioning. The nurse
recognizes that the parents:

A. Are using denial

B. Accept the child’s diagnoses

C. Are using intellectualization

D. Accept their child’s limitation

100. The nurse utilizes play when interacting


with children based on the understanding that
play for the preschool-age child is necessary for
the emotional development of:

A. Projection

B. Introjection

C. Competition

D. Independence

You might also like